Site Loader

Содержание

§ 13. Закон Ома для участка цепи, содержащего ЭДС — ЗФТШ, МФТИ

Пусть на свободные заряды участка цепи `1-2` действуют сторонние силы (силы неэлектростатического происхождения). Тогда говорят, что на участке `1-2` действует электродвижущая сила (ЭДС). За направление действия ЭДС будем считать направление действия сторонних сил на положительные заряды.

Для участка цепи `1-2` можно вывести, используя закон сохранения и превращения энергии, закон Ома для участка цепи, содержащего ЭДС: 

$$ \left(\varphi 1-\varphi 2\right)\pm \mathcal{E}=\pm IR$$. (13.1)

Здесь  $$ \left({\varphi }_{1}-{\varphi }_{2}\right)$$- разность потенциалов (напряжение) между точками `1` и `2`, $$ \mathcal{E}$$ – ЭДС, действующая на участке `1-2`, $$ I$$ – сила тока, $$ R$$ – сопротивление участка `1-2`. В (13.1) величины $$ I$$ и $$ \mathcal{E}$$ взяты положительными, что удобно на практике. При этом справедливо правило знаков: перед $$ \mathcal{E}$$ (или $$ I$$ ) берётся знак `«+»`, если направление действия ЭДС (или направление тока) совпадает с направлением от `1` к `2` и наоборот. Величина $$ IR$$ называется падением напряжения.

На схемах ЭДС на участках цепи обозначается , наличие у участка цепи сопротивления обозначается. . Причём ЭДС и сопротивление могут быть «размазаны» по участку `1-2` произвольным образом и поэтому порядок расположения этих двух символических обозначений для участка цепи в схеме не играет роли. Направление действия ЭДС совпадает с направлением от `«-»` к `«+»` на символическом обозначении.

Следует отметить, что равенство (13.1) справедливо не только для постоянных по времени, $$ I$$, $$ \mathcal{E}$$, $$ R$$ но и для их мгновенных значений.

Рис. 13.1

На участке цепи `1–2`, имеющем сопротивление $$ R=5$$ Ом, идёт ток $$ I=2$$ А и действует ЭДС $$ \mathcal{E}=12$$ В. Найти на участке `1–2` (рис. 13.1) падение напряжения и напряжение.

Падение напряжения есть $$ IR=10$$ B. По закону Ома для участка цепи, содержащего ЭДС, $$ \left({\varphi }_{1}-{\varphi }_{2}\right)-\mathcal{E}=IR$$. Отсюда напряжение

$$ {U}_{12}={\varphi }_{1}-{\varphi }_{2}=IR+\mathcal{E}=22$$ B.

Закон Ома для неоднородного участка цепи

На практике видно, что для поддержания стабильного тока в замкнутой цепи необходимы силы принципиально иной природы, нежели кулоновские, тогда наблюдается случай, когда на участке цепи на свободные электрические заряды одновременно действуют как силы электрического поля, так и сторонние силы (любые неконсервативные силы, действующие на заряд, за исключением сил электрического сопротивления (кулоновских сил)). Такой участок называется неоднородным участком цепи. На рисунке ниже приведен пример такого участка.

Напряженность поля в любой точке цепи равна векторной сумме поля кулоновских сил и поля сторонних сил:

Сформулируем закон Ома для неоднородного участка цепи — Сила тока прямо пропорциональна напряжению на этом участке и обратно пропорциональна его полному сопротивлению:

– формула закона Ома для неоднородного участка цепи.

Где

  • I – сила тока,
  • U12 – напряжение на участке,
  • R – полное сопротивление цепи.

Работа на неоднородном участке цепи

Разность потенциалов характеризует работу силы электрического поля по переносу единичного положительного заряда (q) из точки 1 в точку 2:

— где φ1 и φ 2 – потенциалы на концах участка.

ЭДС характеризует работу сторонних сил по переносу единичного положительного заряда точки 1 в точку 2: — где ε12 – ЭДС, действующая на данном участке, численно равна работе по перемещению единичного положительного заряда вдоль контура.

Напряжение на участке цепи представляет собой суммарную работу сил ЭП и сторонних сил:

Тогда закон Ома примет вид:

ЭДС может быть как положительной, так и отрицательной. Это зависит от полярности включения ЭДС в участок. Если внутри источника тока обход совершается от отрицательного полюса к положительному, то ЭДС положительная (см. рисунок). Сторонние силы при этом совершают положительную работу. Если же обход совершается от положительного полюса к отрицательному, то ЭДС отрицательная. Проще говоря, если ЭДС способствует движению положительных зарядов, то ε>0, иначе ε

Решение задач по закону ому для неоднородного участка цепи

Определить ток, идущий по изображенному на рисунке участку АВ. ЭДС источника 20 В, внутреннее сопротивление 1 Ом, потенциалы точек А и В соответственно 15 В и 5 В, сопротивление проводов 3 Ом.

Дано:Решение:
  • ε = 20 В
  • r = 1 Ом
  • φ1 = 15 В
  • φ2 = 5 В
  • R = 3 Ом
  • Запишем закон Ома для неоднородного участка цепи —
  • Считая, что точка А начало участка, а точка В – конец, возьмем ЭДС со знаком «минус» и, подставив исходные данные, получим
  • Знак «минус» говорит о том, что ток идет от точки В к точке А, от точки с меньшим потенциалом к точке с большим, что обычно для источников тока.
  • Ответ: –2,5 А

Два элемента соединены «навстречу» друг другу, как показано на рисунке. Определить разность потенциалов между точками А и В, если ε1 = 1,4 В, r1 = 0,4 Ом, ε2 = 1,8 В, r2 = 0,6 Ом.

Дано:Решение:
  • ε1 = 1,4 В
  • r1 = 0,4 Ом
  • ε2 = 1,8 В
  • r2 = 0,6 Ом
  • Запишем закон Ома для неоднородного участка цепи —
  • Разобьём схему на два участка: АЕ2В и ВЕ1А. Тогда получим — для первого участка цепи, — для второго участка.
  • Ток на участках один и тот же, то есть можем приравнять правые части уравнений.
  • Знак «минус» показывает, что потенциал точки В выше, чем потенциал точки А.
  • Ответ: -1,56 В.

Закон Ома для участка цепи не содержащего э.д.с.

Напряжение на любом участке электрической цепи, не содержащем ЭДС равно произведению тока, протекающего через участок на сопротивление этого участка.

(рис. 1.9) или (1.11)

Закон Ома для участка цепи, содержащего э.д.с (рис. 1.10)

В общем случае (1.12)

Ток активного участка цепи равен алгебраической сумме его напряжения и ЭДС, деленной на сопротивление участка.

 

Основными законами электрических цепей, наряду с законом Ома, являются законы баланса токов в разветвлениях (первый закон Кирхгофа) и баланса напряжений на замкнутых участках цепи (второй закон Кирхгофа). Законы Кирхгофа

Рассмотрим схему на рисунке 1.11. Пусть заданы значения э.д.с. Е1, Е

4, Е5 и сопротивлений R1, R2, R3, R4, R5;

Необходимо определить: I1, I2, I3, I4, I5, I6.

Рис.1.11

Анализ и расчет разветвленных цепей обычно проводят с помощью законов Кирхгофа.

Первыйзакон Кирхгофа по существу является законом сохранения энергии для электрических цепей.

Алгебраическая сумма токов в ветвях, сходящихся в узле электрическойцепи равна нулю.

(1.13)

Количество уравнений, составляемых по I закону Кирхгофа, на одно меньше общего количества узлов в схеме (узлы выбираем произвольно). При этом ток, входящий в узел, обычно берется со знаком «+», выходящий со знаком «-».

а :

b:

 

Второй закон Кирхгофа применяют к замкнутым контурам.

Алгебраическая сумма ЭДС. в любом контуре электрической цепи постоянного тока равна алгебраической сумме падений напряжений на всех элементах, входящих в этот контур, т. е.:

(1.14)

где n — число источников ЭДС;. m — количество ветвей в контуре.

Токи входят со знаком «+», если их направления совпадают с направлением обхода контура и наоборот.

Для составления уравнений по второму закону Кирхгофа выбирают направление обхода контуров. Если направление тока и обхода совпадают, то произведение IR (падение напряжения) берется со знаком «+», в противном случае со знаком « — ». Если направление ЭДС и обхода совпадают, то ЭДС записывается в уравнение со знаком «+», если не совпадают — то со знаком « — ».Количество уравнений, составляемых по II закону Кирхгофа равно m-(n-1), где m – число неизвестных, n – число узлов.


Узнать еще:

ЛАБОРАТОРНАЯ РАБОТА 24 ИЗУЧЕНИЕ ЗАКОНА ОМА ДЛЯ УЧАСТКА ЦЕПИ, СОДЕРЖАЩЕГО ЭДС. 1. Теоретические основы работы

E — нормальный элемент Вестона.

ЛАБОРАТОРНАЯ РАБОТА 3-7: ИЗМЕРЕНИЕ ЭЛЕКТРОДВИЖУЩИХ СИЛ ГАЛЬВАНИЧЕСКИХ ЭЛЕМЕНТОВ МЕТОДОМ КОМПЕНСАЦИИ Студент группа Допуск Выполнение Защита Цель работы: ознакомление с методами компенсации и применение

Подробнее

ε, r R 1 R 2 С1 «ПОСТОЯННЫЙ ТОК»

С1 «ПОСТОЯННЫЙ ТОК» На рисунке показана электрическая цепь, содержащая источник тока (с отличным от нуля внутренним сопротивлением), два резистора, конденсатор, ключ К, а также амперметр и идеальный вольтметр.

Подробнее

ЛАБОРАТОРНЫЕ РАБОТЫ по курсу физики

Ю. В. Тихомиров ЛАБОРАТОРНЫЕ РАБОТЫ по курсу физики С ЭЛЕМЕНТАМИ КОМПЬЮТЕРНОГО МОДЕЛИРОВАНИЯ ЭЛЕКТРИЧЕСТВО И МАГНЕТИЗМ. ОПТИКА для студентов всех специальностей всех форм обучения МОСКВА — 2012 ЦЕЛЬ РАБОТЫ

Подробнее

ПОСТОЯННЫЙ ЭЛЕКТРИЧЕСКИЙ ТОК

ПОСТОЯННЫЙ ЭЛЕКТРИЧЕСКИЙ ТОК Причины возникновения электрического тока Заряженные объекты являются причиной не только электростатического поля, но еще и электрического тока. В этих двух явлениях, есть

Подробнее

Постоянный электрический ток. Лекция 1

Постоянный электрический ток Лекция 1 Содержание лекции: Электрический ток Уравнение непрерывности Электродвижущая сила 2 Электрический ток Электрический ток упорядоченное движение электрических зарядов

Подробнее

Практическая работа 5

Практическая работа 5 Тема: Расчёт электрических цепей с использованием законов Ома и Кирхгофа. Цель: научиться рассчитывать электрические цепи постоянного тока, используя законы Ома и Кирхгофа. Ход работы

Подробнее

Лабораторная работа 12*

Лабораторная работа 2 ИССЛЕДОВАНИЕ ЭЛЕКТРОСТАТИЧЕСКИХ ПОЛЕЙ Цель работы найти и построить эквипотенциальные поверхности и силовые линии электрического поля между двумя электродами произвольной формы; определить

Подробнее

Законы постоянного тока

Законы постоянного тока Проводники в электростатическом поле E = 0 E = grad φ φ = const S DdS = i q i = 0 Проводники в электростатическом поле Нейтральный проводник, внесенный в электростатическое поле,

Подробнее

ЦЕПИ ПОСТОЯННОГО ТОКА

МИНИСТЕРСТВО ОБРАЗОВАНИЯ И НАУКИ РОССИЙСКОЙ ФЕДЕРАЦИИ КАЗАНСКИЙ ГОСУДАРСТВЕННЫЙ АРХИТЕКТУРНО- СТРОИТЕЛЬНЫЙ УНИВЕРСИТЕТ Кафедра физики ЦЕПИ ПОСТОЯННОГО ТОКА Лабораторная работа 78 Методические указания

Подробнее

Глава 9 Постоянный электрический ток 75

Глава 9 Постоянный электрический ток 75 Электрический ток, сила и плотность тока Электродинамика это раздел электричества, в котором рассматриваются процессы и явления, обусловленные движением электрических

Подробнее

ФИЗИЧЕСКИЕ ВЕЛИЧИНЫ ФОРМУЛЫ

На рисунке показана цепь постоянного тока. Внутренним сопротивлением источника тока можно пренебречь. Установите соответствие между физическими величинами и формулами, по которым их можно рассчитать (

Подробнее

МОСТОВЫЕ МЕТОДЫ ИЗМЕРЕНИЙ

КАЗАНСКАЯ ГОСУДАРСТВЕННАЯ АРХИТЕКТУРНО-СТРОИТЕЛЬНАЯ АКАДЕМИЯ Кафедра физики МЕТОДИЧЕСКИЕ УКАЗАНИЯ К ЛАБОРАТОРНЫМ РАБОТАМ ПО ФИЗИКЕ для студентов специальностей 2903, 2906, 2907, 2908, 2910 Лабораторная

Подробнее

(3.1) (3.2) (3.3) (3.4) η=f(i) P 1, η. 0,5 P 1 =f(i)

Методические указания к выполнению лабораторной работы.1.3 ИССЛЕДОВАНИЕ ЗАВИСИМОСТИ ПОЛЕЗНОЙ МОЩНОСТИ И КОЭФФИЦИЕНТА ПОЛЕЗНОГО ДЕЙСТВИЯ БАТАРЕИ ЭЛЕМЕНТОВ ОТ СИЛЫ ТОКА В ЦЕПИ Филимоненкова Л.В. Электростатика

Подробнее

Глава 1. Основные законы электрической цепи

Глава 1. Основные законы электрической цепи 1.1 Параметры электрической цепи Электрической цепью называют совокупность тел и сред, образующих замкнутые пути для протекания электрического тока. Обычно физические

Подробнее

Лекция 11. Закон Ома

Лекция 11. Закон Ома 11.1. Закон Ома для неоднородного участка цепи. 11.. Закон Ома в дифференциальной форме. 11.3. Работа и мощность. Закон Джоуля Ленца. 11.4. КПД источника тока. 11.5. Закон Кирхгофа.

Подробнее

Тема 3. Постоянный ток

Тема 3. Постоянный ток Лабораторная работа 5 (2.4) Применение закона Ома для расчетов в цепях постоянного тока Введение Георг Ом в 1827 году экспериментально установил закон, согласно которому отношение

Подробнее

Постоянный электрический ток

Постоянный электрический ток Основные определения Электрический ток упорядоченное движение электрических зарядов (носители тока) под действием сил электрического поля. В металлах носителями тока являются

Подробнее

ee m 2 ρ 2 2m U R x = R A. (5) I

Методические указания к выполнению лабораторной работы.1.7 ЭЛЕКТРИЧЕСКОЕ СОПРОТИВЛЕНИЕ МЕТАЛЛОВ Аникин А.И., Фролова Л.Н. Электрическое сопротивление металлов: Методические указания к выполнению лабораторной

Подробнее

Эффект Холла в примесных полупроводниках.

00807. Эффект Холла в примесных полупроводниках. Цель работы: Изучить эффект Холла в примесных полупроводниках. Ознакомиться с методом измерения концентрации и подвижности основных носителей тока в примесных

Подробнее

Закон Ома для активного и пассивного участка линейной электрической цепи

Закон Ома для пассивного участка электрической цепи. 

При протекании электрического тока через сопротивление R, напряжение U и ток I на этом участке связаны между собою согласно закону Ома:

Сопротивление R — это коэффициент пропорциональности между током и напряжением. Чтобы найти сопротивление, нужно напряжение на участке электрической цепи разделить на ток, протекающий на этом же участке.

Закон Ома можно записать через разность потенциалов:

Закон Ома для активного участка электрической цепи.

Закон Ома для активного участка цепи между точками а и в имеет вид:

Напряжение на участке электрической цепи Uab и ЭДС берутся со знаком «плюс», если их направление совпадает с направление протекания тока. Напряжение (разность потенциалов) и источник электродвижущей силы берутся со знаком «минус», если их направление не совпадает с направлением протекания тока.

Пример составления уравнения по закону Ома 

Рассмотрим пример решения задачи на составления уравнения по закону Ома для участка линейной электрической цепи с двумя источниками ЭДС.

Пусть в данной электрической цепи направление тока будет из точки «a» в точку «b». Напряжение Uab Направляется всегда из первой буквы («a») к последней («b»).

Согласно правилу составления уравнения по закону Ома источник ЭДС E1 берем со знаком «плюс», т.к. его направление (направление стрелочки) совпадает с направлением протекающего тока.

Источник ЭДС E2 берем со знаком «минус», т.к. его направление (направление стрелочки) не совпадает с направлением протекающего тока.

Напряжение Uab или разность потенциалов φa — φb берем со знаком «плюс», т.к. его направление совпадает с направление протекающего тока.

Сопротивление R1 и R1 соединены последовательно. При последовательном соединении сопротивлений их эквивалентное значение равно сумме. 

В результате составленное уравнение по закону Ома будет иметь вид:

Пусть потенциал в данной задаче потенциал точки «а» равен 10 вольт, потенциал точки «b» = 7 вольт, E1=25 В, E2=17 В, R1=5 Ом, R2=10 Ом. Рассчитаем величину тока:

 

Полученный ток равен 1 Ампер.

Закон ома устанавливает. Самый главный закон электротехники

Зависит величина воздействия, которое ток может оказывать на проводник, будь то тепловое, химическое или магнитное действие тока . То есть, регулируя силу тока, можно управлять его воздействием. Электрический ток , в свою очередь – это упорядоченное движение частиц под действием электрического поля .

Зависимость силы тока и напряжения

Очевидно, что чем сильнее поле действует на частицы, тем больше будет сила тока в цепи. Электрическое поле характеризуется величиной, называемой напряжением . Следовательно, мы приходит к выводу, что сила тока зависит от напряжения.

И действительно, опытным путем удалось установить, что сила тока связана с напряжением прямо пропорционально. В случаях, когда изменяли величину напряжения в цепи, не меняя всех остальных параметров, сила тока возрастала или уменьшалась во столько же раз, во сколько меняли напряжение.

Связь с сопротивлением

Однако любая цепь или участок цепи характеризуются еще одной немаловажной величиной, называемой сопротивлением электрическому току . Сопротивление связано с силой тока обратно пропорционально. Если на каком-либо участке цепи изменить величину сопротивления, не меняя напряжения на концах этого участка, сила тока также изменится. Причем если мы уменьшим величину сопротивления, то сила тока возрастет во столько же раз. И, наоборот, при увеличении сопротивления сила тока пропорционально уменьшается.

Формула закона Ома для участка цепи

Сопоставив две эти зависимости, можно прийти к такому же выводу, к которому пришел немецкий ученый Георг Ом в 1827 г. Он связал воедино три вышеуказанные физические величины и вывел закон, который назвали его именем. Закон Ома для участка цепи гласит:

Сила тока в участке цепи прямо пропорциональна напряжению на концах этого участка и обратно пропорциональна его сопротивлению.

где I – сила тока,
U – напряжение,
R – сопротивление.

Применение закона Ома

Закон Ома – один из основополагающих законов физики . Открытие его в свое время позволило сделать огромный скачок в науке. В настоящее время невозможно себе представить любой самый элементарный расчет основных электрических величин для любой цепи без использования закона Ома. Представление об этом законе – это не удел исключительно инженеров-электронщиков, а необходимая часть базовых знаний любого мало-мальски образованного человека. Недаром есть поговорка: «Не знаешь закон Ома – сиди дома».

U=IR и R=U/I

Правда, следует понимать, что в собранной цепи величина сопротивления некоторого участка цепи есть величина постоянная, поэтому при изменении силы тока будет изменяться только напряжение и наоборот. Для изменения сопротивления участка цепи следует собрать цепь заново. Расчет же требуемой величины сопротивления при проектировании и сборке цепи можно произвести по закону Ома, исходя из предполагаемых значений силы тока и напряжения, которые будут пропущены через данный участок цепи.

Если изолированный проводник поместить в электрическое поле \(\overrightarrow{E} \), то на свободные заряды \(q\) в проводнике будет действовать сила \(\overrightarrow{F} = q\overrightarrow{E}\) В результате в проводнике возникает кратковременное перемещение свободных зарядов. Этот процесс закончится тогда, когда собственное электрическое поле зарядов, возникших на поверхности проводника, скомпенсирует полностью внешнее поле. Результирующее электростатическое поле внутри проводника будет равно нулю.

Однако, в проводниках при определенных условиях может возникнуть непрерывное упорядоченное движение свободных носителей электрического заряда.

Направленное движение заряженных частиц называется электрическим током.

За направление электрического тока принято направление движения положительных свободных зарядов. Для существования электрического тока в проводнике необходимо создать в нем электрическое поле.

Количественной мерой электрического тока служит сила тока \(I\) — скалярная физическая величина, равная отношению заряда \(\Delta q\), переносимого через поперечное сечение проводника (рис. 1.8.1) за интервал времени \(\Delta t\), к этому интервалу времени:

$$I = \frac{\Delta q}{\Delta t} $$

Если сила тока и его направление не изменяются со временем, то такой ток называется постоянным .

В Международной системе единиц СИ сила тока измеряется в Амперах (А). Единица измерения тока 1 А устанавливается по магнитному взаимодействию двух параллельных проводников с током.

Постоянный электрический ток может быть создан только в замкнутой цепи , в которой свободные носители заряда циркулируют по замкнутым траекториям. Электрическое поле в разных точках такой цепи неизменно во времени. Следовательно, электрическое поле в цепи постоянного тока имеет характер замороженного электростатического поля. Но при перемещении электрического заряда в электростатическом поле по замкнутой траектории, работа электрических сил равна нулю. Поэтому для существования постоянного тока необходимо наличие в электрической цепи устройства, способного создавать и поддерживать разности потенциалов на участках цепи за счет работы сил неэлектростатического происхождения . Такие устройства называются источниками постоянного тока . Силы неэлектростатического происхождения, действующие на свободные носители заряда со стороны источников тока, называются сторонними силами .

Природа сторонних сил может быть различной. В гальванических элементах или аккумуляторах они возникают в результате электрохимических процессов, в генераторах постоянного тока сторонние силы возникают при движении проводников в магнитном поле. Источник тока в электрической цепи играет ту же роль, что и насос, который необходим для перекачивания жидкости в замкнутой гидравлической системе. Под действием сторонних сил электрические заряды движутся внутри источника тока против сил электростатического поля, благодаря чему в замкнутой цепи может поддерживаться постоянный электрический ток.

При перемещении электрических зарядов по цепи постоянного тока сторонние силы, действующие внутри источников тока, совершают работу.

Физическая величина, равная отношению работы \(A_{ст}\) сторонних сил при перемещении заряда \(q\) от отрицательного полюса источника тока к положительному к величине этого заряда, называется электродвижущей силой источника (ЭДС):

$$ЭДС=\varepsilon=\frac{A_{ст}}{q}. $$

Таким образом, ЭДС определяется работой, совершаемой сторонними силами при перемещении единичного положительного заряда. Электродвижущая сила, как и разность потенциалов, измеряется в Вольтах (В).

При перемещении единичного положительного заряда по замкнутой цепи постоянного тока работа сторонних сил равна сумме ЭДС, действующих в этой цепи, а работа электростатического поля равна нулю.

Цепь постоянного тока можно разбить на отдельные участки. Те участки, на которых не действуют сторонние силы (т. е. участки, не содержащие источников тока), называются однородными . Участки, включающие источники тока, называются неоднородными .

При перемещении единичного положительного заряда по некоторому участку цепи работу совершают как электростатические (кулоновские), так и сторонние силы. Работа электростатических сил равна разности потенциалов \(\Delta \phi_{12} = \phi_{1} — \phi_{2}\) между начальной (1) и конечной (2) точками неоднородного участка. Работа сторонних сил равна по определению электродвижущей силе \(\mathcal{E}\), действующей на данном участке. Поэтому полная работа равна

$$U_{12} = \phi_{1} — \phi_{2} + \mathcal{E}$$

Величину U 12 принято называть напряжением на участке цепи 1-2. В случае однородного участка напряжение равно разности потенциалов:

$$U_{12} = \phi_{1} — \phi_{2}$$

Немецкий физик Г. Ом в 1826 году экспериментально установил, что сила тока \(I\), текущего по однородному металлическому проводнику (т. е. проводнику, в котором не действуют сторонние силы), пропорциональна напряжению \(U\) на концах проводника:

$$I = \frac{1}{R} U; \: U = IR$$

где \(R\) = const.

Величину R принято называть электрическим сопротивлением . Проводник, обладающий электрическим сопротивлением, называется резистором . Данное соотношение выражает закон Ома для однородного участка цепи: сила тока в проводнике прямо пропорциональна приложенному напряжению и обратно пропорциональна сопротивлению проводника.

В СИ единицей электрического сопротивления проводников служит Ом (Ом). Сопротивлением в 1 Ом обладает такой участок цепи, в котором при напряжении 1 В возникает ток силой 1 А.

Проводники, подчиняющиеся закону Ома, называются линейными . Графическая зависимость силы тока \(I\) от напряжения \(U\) (такие графики называются вольт-амперными характеристиками , сокращенно ВАХ) изображается прямой линией, проходящей через начало координат. Следует отметить, что существует много материалов и устройств, не подчиняющихся закону Ома, например, полупроводниковый диод или газоразрядная лампа. Даже у металлических проводников при токах достаточно большой силы наблюдается отклонение от линейного закона Ома, так как электрическое сопротивление металлических проводников растет с ростом температуры.

Для участка цепи, содержащего ЭДС, закон Ома записывается в следующей форме:

$$IR = U_{12} = \phi_{1} — \phi_{2} + \mathcal{E} = \Delta \phi_{12} + \mathcal{E}$$
$$\color{blue}{I = \frac{U}{R}}$$

Это соотношение принято называть обобщенным законом Ома или законом Ома для неоднородного участка цепи .

На рис. 1.8.2 изображена замкнутая цепь постоянного тока. Участок цепи (cd ) является однородным.

Рисунок 1.8.2.

Цепь постоянного тока

По закону Ома

$$IR = \Delta\phi_{cd}$$

Участок (ab ) содержит источник тока с ЭДС, равной \(\mathcal{E}\).

По закону Ома для неоднородного участка,

$$Ir = \Delta \phi_{ab} + \mathcal{E}$$

Сложив оба равенства, получим:

$$I(R+r) = \Delta\phi_{cd} + \Delta \phi_{ab} + \mathcal{E}$$

Но \(\Delta\phi_{cd} = \Delta \phi_{ba} = -\Delta \phi_{ab}\).

$$\color{blue}{I=\frac{\mathcal{E}}{R + r}}$$

Эта формула выражает закон Ома для полной цепи : сила тока в полной цепи равна электродвижущей силе источника, деленной на сумму сопротивлений однородного и неоднородного участков цепи (внутреннего сопротивления источника).

Сопротивление r неоднородного участка на рис. 1.8.2 можно рассматривать как внутреннее сопротивление источника тока . В этом случае участок (ab ) на рис. 1.8.2 является внутренним участком источника. Если точки a и b замкнуть проводником, сопротивление которого мало по сравнению с внутренним сопротивлением источника (\(R\ \ll r\)), тогда в цепи потечет ток короткого замыкания

$$I_{кз}=\frac{\mathcal{E}}{r}$$

Сила тока короткого замыкания — максимальная сила тока, которую можно получить от данного источника с электродвижущей силой \(\mathcal{E}\) и внутренним сопротивлением \(r\). У источников с малым внутренним сопротивлением ток короткого замыкания может быть очень велик и вызывать разрушение электрической цепи или источника. Например, у свинцовых аккумуляторов, используемых в автомобилях, сила тока короткого замыкания может составлять несколько сотен ампер. Особенно опасны короткие замыкания в осветительных сетях, питаемых от подстанций (тысячи ампер). Чтобы избежать разрушительного действия таких больших токов, в цепь включаются предохранители или специальные автоматы защиты сетей.

В ряде случаев для предотвращения опасных значений силы тока короткого замыкания к источнику последовательно подсоединяется некоторое внешнее сопротивление. Тогда сопротивление r равно сумме внутреннего сопротивления источника и внешнего сопротивления, и при коротком замыкании сила тока не окажется чрезмерно большой.

Если внешняя цепь разомкнута, то \(\Delta \phi_{ba} = -\Delta \phi_{ab} = \mathcal{E}\), т. е. разность потенциалов на полюсах разомкнутой батареи равна ее ЭДС.

Если внешнее нагрузочное сопротивление R включено и через батарею протекает ток I , разность потенциалов на ее полюсах становится равной

$$\Delta \phi_{ba} = \mathcal{E} — Ir$$

На рис. 1.8.3 дано схематическое изображение источника постоянного тока с ЭДС равной \(\mathcal{E}\) и внутренним сопротивлением r в трех режимах: «холостой ход», работа на нагрузку и режим короткого замыкания (к. з.). Указаны напряженность \(\overrightarrow{E}\) электрического поля внутри батареи и силы, действующие на положительные заряды:\(\overrightarrow{F}_{э}\) — электрическая сила и \(\overrightarrow{F}_{ст}\) — сторонняя сила. В режиме короткого замыкания электрическое поле внутри батареи исчезает.

Для измерения напряжений и токов в электрических цепях постоянного тока используются специальные приборы — вольтметры и амперметры .

Вольтметр предназначен для измерения разности потенциалов, приложенной к его клеммам. Он подключается параллельно участку цепи, на котором производится измерение разности потенциалов. Любой вольтметр обладает некоторым внутренним сопротивлением \(R_{В}\). Для того, чтобы вольтметр не вносил заметного перераспределения токов при подключении к измеряемой цепи, его внутреннее сопротивление должно быть велико по сравнению с сопротивлением того участка цепи, к которому он подключен. Для цепи, изображенной на рис. 1.8.4, это условие записывается в виде:

$$R_{В} \gg R_{1}$$

Это условие означает, что ток \(I_{В} = \Delta \phi_{cd} / R_{В}\), протекающий через вольтметр, много меньше тока \(I = \Delta \phi_{cd} / R_{1}\), который протекает по тестируемому участку цепи.

Поскольку внутри вольтметра не действуют сторонние силы, разность потенциалов на его клеммах совпадает по определению с напряжением. Поэтому можно говорить, что вольтметр измеряет напряжение.

Амперметр предназначен для измерения силы тока в цепи. Амперметр включается последовательно в разрыв электрической цепи, чтобы через него проходил весь измеряемый ток. Амперметр также обладает некоторым внутренним сопротивлением \(R_{А}\). В отличие от вольтметра, внутреннее сопротивление амперметра должно быть достаточно малым по сравнению с полным сопротивлением всей цепи. Для цепи на рис. 1.8.4 сопротивление амперметра должно удовлетворять условию

$$R_{А} \ll (r + R_{1} + R{2})$$

чтобы при включении амперметра ток в цепи не изменялся.

Измерительные приборы — вольтметры и амперметры — бывают двух видов: стрелочные (аналоговые) и цифровые. Цифровые электроизмерительные приборы представляют собой сложные электронные устройства. Обычно цифровые приборы обеспечивают более высокую точность измерений.

В 1826 году немецкий ученый Георг Ом совершил открытие и описал
эмпирический закон о соотношении между собой таких показателей как сила тока, напряжение и особенности проводника в цепи. Впоследствии, по имени ученого он стал называться закон Ома.

В дальнейшем выяснилось, что эти особенности не что иное, как сопротивление проводника, возникающее в процессе его контакта с электричеством. Это внешнее сопротивление (R). Есть также внутреннее сопротивление (r), характерное для источника тока.

Закон Ома для участка цепи

Согласно обобщенному закону Ома для некоторого участка цепи, сила тока на участке цепи прямо пропорциональна напряжению на концах участка и обратно пропорциональна сопротивлению.

Где U – напряжение концов участка,I– сила тока, R– сопротивление проводника.

Беря во внимание вышеприведенную формулу, есть возможность найти неизвестные значенияUиR, сделав несложные математические операции.

Данные выше формулы справедливы лишь когда сеть испытывает на себе одно сопротивление.

Закон Ома для замкнутой цепи

Сила тока полной цепи равна ЭДС, деленной на сумму сопротивлений однородного и неоднородного участков цепи.

Замкнутая сеть имеет одновременно сопротивления внутреннего и внешнего характера. Поэтому формулы отношения будут уже другими.

Где E – электродвижущая сила (ЭДС), R- внешнее сопротивление источника, r-внутреннее сопротивление источника.

Закон Ома для неоднородного участка цепи

Замкнутая электрическая сеть содержит участки линейного и нелинейного характера. Участки, не имеющие источника тока и не зависящие от стороннего воздействия являются линейными, а участки, содержащие источник – нелинейными.

Закон Ома для участка сети однородного характера был изложен выше. Закон на нелинейном участке будет иметь следующий вид:

I = U/ R = f1 – f2 + E/ R

Где f1 – f2 – разница потенциалов на конечных точках рассматриваемого участка сети

R – общее сопротивление нелинейного участка цепи

ЭДС нелинейного участка цепи бывает больше нуля или меньше. Если направление движения тока, идущего из источника с движением тока в электрической сети, совпадают, будет преобладать движение зарядов положительного характера и ЭДС будет положительная. В случае же совпадения направлений, в сети будет увеличено движение отрицательных зарядов, создаваемых ЭДС.

Закон Ома для переменного тока

При имеющейся в сети емкости или инертности, необходимо учитывать при проводимых вычислениях, что они выдают свое сопротивление, от действия которого ток приобретает переменный характер.

Закон Ома для переменного тока выглядит так:

где Z – сопротивление по всей длине электрической сети. Его еще называют импеданс. Импеданс составляют сопротивления активного и реактивного характера.

Закон Ома не является основным научным законом, а лишь эмпирическим отношением, причем в некоторых условиях оно может не соблюдаться:

  • Когда сеть обладает высокой частотой, электромагнитное поле меняется с большой скоростью, и при расчетах необходимо учитывать инертность носителей заряда;
  • В условиях низкой температуры с веществами, которые обладают сверхпроводимостью;
  • Когда проводник сильно нагревается проходящим напряжением, отношение тока к напряжению становится переменным и может не соответствовать общему закону;
  • При нахождении под высоким напряжением проводника или диэлектрика;
  • В светодиодных лампах;
  • В полупроводниках и полупроводниковых приборах.

В свою очередь элементы и проводники, соблюдающие закон Ома, называются омическими.

Закон Ома может дать объяснение некоторым явлениям природы. Например, когда мы видим птиц, сидящих на высоковольтных проводах, у нас возникает вопрос – почему на них не действует электрический ток? Объясняется это довольно просто. Птицы, сидя на проводах, представляют собой своеобразные проводники. Большая часть напряжения приходится на промежутки между птицами, а та доля, что приходится на сами «проводники» не представляет для них опасности.

Но это правило работает лишь при единичном соприкосновении. Если птица заденет клювом или крылом провод или телеграфный столб, она неминуемо погибнет от огромного количества напряжения, которое несут в себе эти участки. Такие случаи происходят повсеместно. Поэтому в целях безопасности в некоторых населенных пунктах установлены специальные приспособления, защищающие птиц от опасного напряжения. На таких насестах птицы находятся в полной безопасности.

Закон Ома также широко применятся на практике. Электричество смертельно опасно для человека при одном лишь касании к оголенному проводу. Но в некоторых случаях сопротивление человеческого тела может быть разным.

Так, например, сухая и неповрежденная кожа обладает большим сопротивлением к воздействию электричества нежели рана или кожа, покрытая потом. В следствие переутомления, нервного напряжения и опьянения, даже при небольшом напряжении тока человек может получить сильный удар током.

В среднем, сопротивление тела человека – 700 Ом, значит, для человека является безопасным напряжение в 35 В. Работая с большим напряжением, специалисты используют .

Закон Ома является основным законом, который используют при расчетах цепей постоянного тока. Он является фундаментальным и может применяться для любых физических систем, где есть потоки частиц и поля, преодолевается сопротивление.

Законы или правила Кирхгофа являются приложением к закону Ома, используемым для расчета сложных электрических цепей постоянного тока.

Закон Ома

Обобщенный закон Ома для неоднородного участка цепи (участка цепи, содержащего источник ЭДС) имеет вид:

Разность потенциалов на концах участка цепи; — ЭДС источника на рассматриваемом участке цепи; R — внешнее сопротивление цепи; r — внутреннее сопротивление источника ЭДС. Если цепь разомкнута, значит, тока в ней нет (), то из (2) получим:

ЭДС, действующая в незамкнутой цепи, равна разности потенциалов на ее концах. Получается, для нахождения ЭДС источника следует измерить разность потенциалов на его клеммах при незамкнутой цепи.

Закон Ома для замкнутой цепи записывают как:

Величину иногда называют полным сопротивлением цепи. Формула (2) показывает, что электродвижущая сила источника тока, деленная на полное сопротивление равна силе тока в цепи.

Закон Кирхгофа

Пусть имеется произвольная разветвленная сеть проводников. В отдельных участках включены разнообразные источники тока. ЭДС источников постоянны и будем считать известными. При этом токи во всех участках цепи и разности потенциалов на них можно вычислить при помощи закона Ома и закона сохранения заряда.

Для упрощения решения задач по расчетам разветвлённых электрических цепей, имеющих несколько замкнутых контуров, несколько источников ЭДС, используют законы (или правила) Кирхгофа. Правила Кирхгофа служат для того, чтобы составить систему уравнений, из которой находят силы тока в элементах сложной разветвленной цепи.

Первый закон Кирхгофа

Сумма токов в узле цепи с учетом их знаков равна нулю:

Первое правило Кирхгофа является следствием закона сохранения электрического заряда. Алгебраическая сумма токов, сходящихся в любом узле цепи — это заряд, который приходит в узел за единицу времени.

При составлении уравнение используя законы Кирхгофа важно учитывать знаки с которыми силы токов входят в эти уравнения. Следует считать, что токи, идущие к точке разветвления, и исходящие от разветвления имеют противоположные знаки. При этом нужно для себя определить какое направление (к узлу или от узла) считать положительным.

Второй закон Кирхгофа

Произведение алгебраической величины силы тока (I) на сумму вешних и внутренних сопротивлений всех участков замкнутого контура равно сумме алгебраических значений сторонних ЭДС () рассматриваемого контура:

Каждое произведение определяет разность потенциалов, которая существовала бы между концами соответствующего участка, если бы ЭДС в нем была равно нулю. Величину называют падением напряжения, которое вызывается током.

Второй закон Кирхгофа иногда формулируют следующим образом:

Для замкнутого контура сумма падений напряжения есть сума ЭДС в рассматриваемом контуре.

Второе правило (закон) Кирхгофа является следствием обобщенного закона Ома. Так, если в изолированной замкнутой цепи есть один источник ЭДС, то сила тока в цепи будет такой, что сумма падения напряжения на внешнем сопротивлении и внутреннем сопротивлении источника будет равна сторонней ЭДС источника. Если источников ЭДС несколько, то берут их алгебраическую сумму. Знак ЭДС выбирается положительным, если при движении по контуру в положительном направлении первым встречается отрицательный полюс источника. (За положительное направление обхода контура принимают направление обхода цепи либо по часовой стрелке, либо против нее).

Примеры решения задач

ПРИМЕР 1

ЗаданиеВольтметр включили последовательно в цепь с сопротивлением, равным , при этом прибор показал напряжение . Сопротивление заменили на . При этом показания вольтметра изменились, и напряжение на вольтметре стало . Каково сопротивление , если сопротивление вольтметра равно r?


РешениеПо закону Ома сила тока, которая течет через вольтметр и сопротивление равна (в первом случае рис.1(а)):

Во втором случае:

Сила тока в любом месте цепи рис.1(а) равна , следовательно, напряжение, которое показывает вольтметр в первом случае равно:

Из (1.3), получим:

Во втором случае, имеем:

Приравняем левые части выражений (1.4) и (1.5):

Из формулы (1.6), выразим искомое сопротивление:

Реферат

Закон Ома. История открытия. Различные виды закона Ома.

1. Общий вид закона Ома.

2. История открытия закона Ома, краткая биография ученого.

3. Виды законов Ома.

Закон Ома устанавливает зависи­мость между силой тока I в проводнике и разностью потенциалов (напряже­нием) U между двумя фиксированными точками (сечениями) этого проводника:

(1) Коэффициент пропорциональности R , завися­щий от геометрических и электрических свойств проводника и от температуры, называется омическим сопротивлением или просто сопротивлением данного участка проводника. Закон Ома был от­крыт в 1826 нем. физиком Г. Омом.

Георг Симон Ом родился 16 марта 1787 года в Эрлангене, в семье потомственного слесаря. После окончания школы Георг поступил в городскую гимназию. Гимназия Эрлангена курировалась университетом. Занятия в гимназии вели четыре профессора. Георг, закончив гимназию, весной 1805 года приступил к изучению математики, физики и философии на философском факультете Эрлангенского университета.

Проучившись три семестра, он принял приглашение занять место учителя математики в частной школе швейцарского городка Готтштадта.

В 1811 году он возвращается в Эрланген, заканчивает университет и получает степень доктора философии. Сразу же по окончании университета ему была предложена должность приват-доцента кафедры математики этого же университета.

В 1812 году Ом был назначен учителем математики и физики школы в Бамберге. В 1817 году он публикует свою первую печатную работу, посвященную методике преподавания «Наиболее оптимальный вариант преподавания геометрии в подготовительных классах». Ом занялся исследованиями электричества. В основу своего электроизмерительного прибора Ом заложил конструкцию крутильных весов Кулона. Результаты своих исследований Ом оформил в виде статьи под названием «Предварительное сообщение о законе, по которому металлы проводят контактное электричество». Статья была опубликована в 1825 году в «Журнале физики и химии», издаваемом Швейггером. Однако выражение, найденное и опубликованное Омом, оказалось неверным, что стало одной из причин его длительного непризнания. Приняв все меры предосторожности, заранее устранив все предполагаемые источники ошибок, Ом приступил к новым измерениям.

Появляется в свет его знаменитая статья «Определение закона, по которому металлы проводят контактное электричество, вместе с наброском теории вольтаического аппарата и мультипликатора Швейггера», вышедшая в 1826 году в «Журнале физики и химии».

В мае 1827 года «Теоретические исследования электрических цепей» объемом в 245 страниц, в которых содержались теперь уже теоретические рассуждения Ома по электрическим цепям. В этой работе ученый предложил характеризовать электрические свойства проводника его сопротивлением и ввел этот термин в научный обиход. Ом нашел более простую формулу для закона участка электрической цепи, не содержащего ЭДС: «Величина тока в гальванической цепи прямо пропорциональна сумме всех напряжений и обратно пропорциональна сумме приведенных длин. При этом общая приведенная длина определяется как сумма всех отдельных приведенных длин для однородных участков, имеющих различную проводимость и различное поперечное сечение».

В 1829 году появляется его статья «Экспериментальное исследование работы электромагнитного мультипликатора», в которой были заложены основы теории электроизмерительных приборов. Здесь же Ом предложил единицу сопротивления, в качестве которой он выбрал сопротивление медной проволоки длиной 1 фут и поперечным сечением в 1 квадратную линию.

В 1830 году появляется новое исследование Ома «Попытка создания приближенной теории униполярной проводимости».

Только в 1841 году работа Ома была переведена на английский язык, в 1847 году — на итальянский, в 1860 году — на французский.

16 февраля 1833 года, через семь лет после выхода из печати статьи, в которой было опубликовано его открытие, Ому предложили место профессора физики во вновь организованной политехнической школе Нюрнберга. Ученый приступает к исследованиям в области акустики. Результаты своих акустических исследований Ом сформулировал в виде закона, получившего впоследствии название акустического закона Ома.

Раньше всех из зарубежных ученых закон Ома признали русские физики Ленц и Якоби. Они помогли и его международному признанию. При участии русских физиков, 5 мая 1842 года Лондонское Королевское общество наградило Ома золотой медалью и избрало своим членом.

В 1845 году его избирают действительным членом Баварской академии наук. В 1849 году ученого приглашают в Мюнхенский университет на должность экстраординарного профессора. В этом же году он назначается хранителем государственного собрания физико-математических приборов с одновременным чтением лекций по физике и математике. В 1852 году Ом получил должность ординарного профессора. Ом скончался 6 июля 1854 года. В 1881 году на электротехническом съезде в Париже ученые единогласно утвердили название единицы сопротивления — 1 Ом.

В общем случае зависимость между I и U нелинейна, однако на практике всегда можно в определенном интервале напряжений считать её линейной и применять закон Ома; для металлов и их сплавов этот интервал практически неограничен.

Закон Ома в форме (1) справедлив для участков цепи, не содержащих источ­ников ЭДС. При наличии таких источников (аккумуляторов, термопар, ге­нераторов и т. д.) закон Ома имеет вид:

(2) — ЭДС всех источников, вклю­чённых в рассматриваемый участок цепи. Для замкнутой цепи закон Ома при­нимает вид: (3) — полное сопротивление цепи, равное сумме внешнего сопротив­ления r и внутреннего сопротивления источника ЭДС. Обобщением закона Ома на случай разветвлённой цепи является правило 2-е Кирхгофа.

Закон Ома можно записать в дифференциальной форме, связывающей в каждой точке проводника плотность тока j с полной напряжённостью электрического поля. Потенциальное. электрическое поле напряжённости Е , создаваемое в проводниках микроскопическими зарядами (электронами, ионами) самих проводников, не может поддерживать стационарное движение свободных зарядов (ток), т. к. работа этого поля на замкнутом пути равна нулю. Ток поддерживается неэлектростатическими силами различного происхождения (индукционного, химического, теплового и т.д.), которые действуют в источниках ЭДС и которые можно представить в виде некоторого эквивалентного непотенциального поля с напряженностью E СТ, называемого сторонним. Полная напряженность поля, действующего внутри проводника на заряды, в общем случае равна E + E СТ . Соответственно, дифференциальный закон Ома имеет вид:

или , (4) — удельное сопротивление материала проводника, а — его удельная электропроводность.

Закон Ома в комплексной форме справедлив также для синусоидальных квазистационарных токов.

Теория закон ома. Все виды законов ома

Электрический ток и опасное напряжение невозможно услышать (за исключением гудящих высоковольтных линий и электроустановок). Токоведущие части, находящиеся под напряжением, ничем не отличаются по внешнему виду.

Невозможно узнать их и по запаху, и повышенной температурой в штатных режимах работы они не отличаются. Но включаем в безмолвную и тихую розетку пылесос, щелкаем выключателем — и энергия словно берется из ниоткуда, сама по себе, материализуясь в виде шума и компрессии внутри бытового прибора.

Опять же, если мы воткнем в разъемы розетки два гвоздя и возьмемся за них, то буквально всем своим телом ощутим реальность и объективность существования электрического тока. Делать это, конечно, настоятельно не рекомендуется. Но примеры с пылесосом и гвоздями наглядно демонстрируют нам, что изучение и понимание основных законов электротехники способствует безопасности при обращении с бытовым электричеством, а также устранению суеверных предубеждений, связанных с электрическим током и напряжением.

Итак, рассмотрим один, самый ценный закон электротехники, который полезно знать. И попытаемся сделать это в как можно более популярной форме.

Закон Ома

1. Дифференциальная форма записи закона Ома

Самый главный закон электротехники — это, конечно, закон Ома . О его существовании знают даже люди, не имеющие отношения к электротехнике. Но между тем вопрос «А знаешь ли ты закон Ома?» в технических ВУЗах является ловушкой для зарвавшихся и самонадеянных школяров. Товарищ, разумеется, отвечает, что закон Ома знает отлично, и тогда к нему обращаются с просьбой привести этот закон в дифференциальной форме. Тут-то и выясняется, что школяру или первокурснику еще учиться и учиться.

Однако дифференциальная форма записи закона Ома на практике почти неприменима. Она отражает зависимость между плотностью тока и напряженностью поля:

где G — это проводимость цепи; Е — напряженность электрического тока.

Все это — попытки выразить электрический ток, принимая во внимание только физические свойства материала проводника, без учета его геометрических параметров (длина, диаметр и тому подобное). Дифференциальная форма записи закона Ома — это чистая теория, знание ее в быту совершенно не требуется.

2. Интегральная форма записи закона Ома для участка цепи

Иное дело — интегральная форма записи. Она тоже имеет несколько разновидностей. Самой популярной из них является закон Ома для участка цепи: I=U/R

Говоря по-другому, ток в участке цепи всегда тем выше, чем больше приложенное к этому участку напряжение и чем меньше сопротивление этого участка.

Вот этот «вид» закона Ома просто обязателен к запоминанию для всех, кому хоть иногда приходится иметь дело с электричеством. Благо, и зависимость-то совсем простая. Ведь напряжение в сети можно считать неизменным. Для розетки оно равно 220 вольт. Поэтому получается, что ток в цепи зависит только от сопротивления цепи, подключаемой к розетке. Отсюда простая мораль: за этим сопротивлением надо следить.

Короткие замыкания, которые у всех на слуху, случаются именно по причине низкого сопротивления внешней цепи. Предположим, что из-за неправильного соединения проводов в ответвительной коробке фазный и нулевой провода оказались напрямую соединены между собой. Тогда сопротивление участка цепи резко снизится практически до нуля, а ток так же резко возрастет до очень большой величины. Если электропроводка выполнена правильно, то сработает автоматический выключатель, а если его нет, или он неисправен или подобран неправильно, то провод не справится с возросшим током, нагреется, расплавится и, возможно, вызовет пожар.

Но бывает, что приборы, включенные в розетку и отработавшие уже далеко не один час, становятся причиной короткого замыкания. Типичный случай — вентилятор, обмотки двигателя которого подверглись перегреву из-за заклинивания лопастей. Изоляция обмоток двигателя не рассчитана на серьезный нагрев, она быстро приходит в негодность. В результате появляются межвитковые короткие замыкания, которые снижают сопротивление и, в соответствии с законом Ома, также ведут к увеличению тока.

Повышенный ток, в свою очередь, приводит изоляцию обмоток в полную негодность, и наступает уже не межвитковое, а самое настоящее, полноценное короткое замыкание. Ток идет помимо обмоток, сразу из фазного в нулевой провод. Правда, все сказанное может случиться только с совсем простым и дешевым вентилятором, не оборудованным тепловой защитой.

Закон Ома для переменного тока

Надо отметить, что приведенная запись закона Ома описывает участок цепи с постоянным напряжением. В сетях переменного напряжения существует дополнительное реактивное сопротивление, а полное сопротивление приобретает значение квадратного корня из суммы квадратов активного и реактивного сопротивления.

Закон Ома для участка цепи переменного тока принимает вид: I=U/Z ,

где Z — полное сопротивление цепи.

Но большое реактивное сопротивление свойственно, прежде всего, мощным электрическим машинам и силовой преобразовательной технике. Внутреннее электрическое сопротивление бытовых приборов и светильников практически полностью является активным. Поэтому в быту для расчетов можно пользоваться самой простой формой записи закона Ома: I=U/R.

3. Интегральная форма записи для полной цепи

Раз есть форма записи закона для участка цепи, то существует и закон Ома для полной цепи: I=E/(r+R) .

Здесь r — внутреннее сопротивление источника ЭДС сети, а R — полное сопротивление самой цепи.

За физической моделью для иллюстрации этого подвида закона Ома далеко ходить не надо — это бортовая электрическая сеть автомобиля, аккумулятор в которой является источником ЭДС. Нельзя считать, что сопротивление аккумулятора равно абсолютному нулю, поэтому даже при прямом замыкании между его клеммами (отсутствии сопротивления R) ток вырастет не до бесконечности, а просто до высокого значения. Однако этого высокого значения, конечно, хватит для того, чтобы вызвать расплавление проводов и возгорание обшивки авто. Поэтому электрические цепи автомобилей защищают от короткого замыкания при помощи предохранителей.

Такой защиты может оказаться недостаточно, если замыкание произойдет до блока предохранителей относительно аккумулятора, или если вовсе один из предохранителей заменен на кусок медной проволоки. Тогда спасение только в одном — необходимо как можно быстрее разорвать цепь полностью, откинув «массу», то есть минусовую клемму.

4. Интегральная форма записи закона Ома для участка цепи, содержащего источник ЭДС

Следует упомянуть и о том, что есть и еще одна разновидность закона Ома — для участка цепи, содержащего источник ЭДС:

Здесь U — это разность потенциалов в начале и в окончании рассматриваемого участка цепи. Знак перед величиной ЭДС зависит от направленности ее относительно напряжения. Воспользоваться законом Ома для участка цепи нередко приходится при определении параметров цепи, когда часть схемы недоступна для детального изучения и не интересует нас. Допустим, она скрыта неразъемными деталями корпуса. В оставшейся схеме имеется источник ЭДС и элементы с известным сопротивлением. Тогда, замерив напряжение на входе неизвестной части схемы, можно вычислить ток, а после этого — и сопротивление неизвестного элемента.

Выводы

Таким образом, мы можем увидеть, что «простой» закон Ома далеко не так прост, как кому-то, возможно, казалось. Зная все формы интегральной записи законов Ома, можно понять и легко запомнить многие требования электробезопасности, а также приобрести уверенность в обращении с электричеством.

Основным законом электротехники, при помощи которого можно изучать и рассчитывать электрические цепи, является закон Ома, устанавливающий соотношение между током, напряжением и сопротивлением. Необходимо отчетливо понимать его сущность и уметь правильно пользоваться им при решении практических задач. Часто в электротехнике допускаются ошибки из-за неумения правильно применить закон Ома.

Закон Ома для участка цепи гласит: ток прямо пропорционален напряжению и обратно пропорционален сопротивлению.

Если увеличить в несколько раз напряжение, действующее в электрической цепи, то ток в этой цепи увеличится во столько же раз. А если увеличить в несколько раз сопротивление цепи, то ток во столько же раз уменьшится. Подобно этому водяной поток в трубе тем больше, чем сильнее давление и чем меньше сопротивление, которое оказывает труба движению воды.

В популярной форме этот закон можно сформулировать следующим образом: чем выше напряжение при одном и том же сопротивлении, тем выше сила тока и в то же время чем выше сопротивление при одном и том же напряжении, тем ниже сила тока.

Чтобы выразить закон Ома математически наиболее просто, считают, что сопротивление проводника, в котором при напряжении 1 В проходит ток 1 А, равно 1 Ом.

Ток в амперах можно всегда определить, если разделить напряжение в вольтах на сопротивление в омах. Поэтому закон Ома для участка цепи записывается следующей формулой:

I = U/R.

Магический треугольник

Любой участок или элемент электрической цепи можно охарактеризовать при помощи трёх характеристик: тока, напряжения и сопротивления.

Как использовать треугольник Ома: закрываем искомую величину — два других символа дадут формулу для её вычисления. Кстати, законом Ома называется только одна формула из треугольника – та, которая отражает зависимость тока от напряжения и сопротивления. Две другие формулы, хотя и являются её следствием, физического смысла не имеют.

Расчеты, выполняемые с помощью закона Ома для участка цепи, будут правильны в том случае, когда напряжение выражено в вольтах, сопротивление в омах и ток в амперах. Если используются кратные единицы измерений этих величин (например, миллиампер, милливольт, мегаом и т. д.), то их следует перевести соответственно в амперы, вольты и омы. Чтобы подчеркнуть это, иногда формулу закона Ома для участка цепи пишут так:

ампер = вольт/ом

Можно также рассчитывать ток в миллиамперах и микроамперах, при этом напряжение должно быть выражено в вольтах, а сопротивление — в килоомах и мегаомах соответственно.

Другие статьи про электричество в простом и доступном изложении:

Расчет напряжения с помощью закона Ома можно показать на следующем примере. Пусть через участок цепи с сопротивлением 10 кОм проходит ток 5 мА и требуется определить напряжение на этом участке.

Умножив I = 0,005 А на R -10000 Ом, получим напряжение,равное 5 0 В. Можно было бы получить тот же результат, умножив 5 мА на 10 кОм: U = 50 В

В электронных устройствах ток обычно выражается в миллиамперах, а сопротивление — в килоомах. Поэтому удобно в расчетах по закону Ома применять именно эти единицы измерений.

По закону Ома рассчитывается также сопротивление, если известно напряжение и ток. Формула для этого случая пишется следующим образом: R = U/I.

Сопротивление всегда представляет собой отношение напряжения к току. Если напряжение увеличить или уменьшить в несколько раз, то ток увеличится или уменьшится в такое же число раз. Отношение напряжения к току, равное сопротивлению, остается неизменным.

Не следует понимать формулу для определения сопротивления в том смысле, что сопротивление данного проводника зависит оттока и напряжения. Известно, что оно зависит от длины, площади сечения и материала проводника. По внешнему виду формула для определения сопротивления напоминает формулу для расчета тока, но между ними имеется принципиальная разница.

Ток в данном участке цепи действительно зависит от напряжения и сопротивления и изменяется при их изменении. А сопротивление данного участка цепи является величиной постоянной, не зависящей от изменения напряжения и тока, но равной отношению этих величин.

Когда один и тот же ток проходит в двух участках цепи, а напряжения, приложенные к ним, различны, то ясно, что участок, к которому приложено большее напряжение, имеет соответственно большее сопротивление.

А если под действием одного и того же напряжения в двух разных участках цепи проходит различный ток, то меньший ток всегда будет на том участке, который имеет большее сопротивление. Все это вытекает из основной формулировки закона Ома для участка цепи, т. е. из того, что ток тем больше, чем больше напряжение и чем меньше сопротивление.

Расчет сопротивления с помощью закона Ома для участка цепи покажем на следующем примере. Пусть требуется найти сопротивление участка, через который при напряжении 40 В проходит ток 50 мА. Выразив ток в амперах, получим I = 0,05 А. Разделим 40 на 0,05 и найдем, что сопротивление составляет 800 Ом.

Закон Ома можно наглядно представить в виде так называемой вольт-амперной характеристики . Как известно, прямая пропорциональная зависимость между двумя величинами представляет собой прямую линию, проходящую через начало координат. Такую зависимость принято называть линейной .

На рис. 2 показан в качестве примера график закона Ома для участка цепи с сопротивлением 100 Ом. По горизонтальной оси отложено напряжение в вольтах, а по вертикальной оси — ток в амперах. Масштаб тока и напряжения может быть выбран каким угодно. Прямая линия проведена так, что для любой ее точки отношение напряжения к току равно 100 Ом. Например, если U = 50 В, то I = 0,5 А и R = 50: 0,5 = 100 Ом.

Рис. 2 . Закон Ома (вольт-амперная характеристика)

График закона Ома для отрицательных значений тока и напряжения имеет такой же вид. Это говорит о том, что ток в цепи проходит одинаково в обоих направлениях. Чем больше сопротивление, тем меньше получается ток при данном напряжении и тем более полого идет прямая.

Приборы, у которых вольт-амперная характеристика является прямой линией, проходящей через начало координат, т. е. сопротивление остается постоянным при изменении напряжения или тока, называются линейными приборами . Применяют также термины линейные цепи, линейные сопротивления.

Существуют также приборы, у которых сопротивление изменяется при изменении напряжения или тока. Тогда зависимость между током и напряжением выражается не по закону Ома, а более сложно. Для таких приборов вольт-амперная характеристика не будет прямой линией, проходящей через начало координат, а является либо кривой, либо ломаной линией. Эти приборы называются нелинейными .

Мнемоническая диаграмма для закона Ома

Георг Симон Ом начал свои исследования вдохновляясь знаменитым трудом Жана Батиста Фурье «Аналитическая теория тепла». В этой работе Фурье представлял тепловой поток между двумя точками как разницу температур, а изменение теплового потока связывал с его прохождением через препятствие неправильной формы из теплоизолирующего материала. Аналогично этому Ом обуславливал возникновение электрического тока разностью потенциалов.

Исходя из этого Ом стал экспериментировать с разными материалами проводника. Для того, чтобы определить их проводимость он подключал их последовательно и подгонял их длину таким образом, чтобы сила тока была одинаковой во всех случаях.

Важно при таких измерениях было подбирать проводники одного и того же диаметра. Ом, замеряя проводимость серебра и золота, получил результаты, которые по современным данным не отличаются точностью. Так, серебряный проводник у Ома проводил меньше электрического тока, чем золотой. Сам Ом объяснял это тем, что его проводник из серебра был покрыт маслом и из-за этого, по всей видимости, опыт не дал точных результатов.

Однако не только с этим были проблемы у физиков, которые в то время занимались подобными экспериментами с электричеством. Большие трудности с добычей чистых материалов без примесей для опытов, затруднения с калибровкой диаметра проводника искажали результаты тестов. Еще большая загвоздка состояла в том, что сила тока постоянно менялась во время испытаний, поскольку источником тока служили переменные химические элементы. В таких условиях Ом вывел логарифмическую зависимость силы тока от сопротивления провода.

Немногим позже немецкий физик Поггендорф, специализировавшийся на электрохимии, предложил Ому заменить химические элементы на термопару из висмута и меди. Ом начал свои эксперименты заново. В этот раз он пользовался термоэлектрическим устройством, работающем на эффекте Зеебека в качестве батареи. К нему он последовательно подключал 8 проводников из меди одного и того же диаметра, но различной длины. Чтобы измерить силу тока Ом подвешивал с помощью металлической нити над проводниками магнитную стрелку. Ток, шедший параллельно этой стрелке, смещал ее в сторону. Когда это происходило физик закручивал нить до тех пор, пока стрелка не возвращалась в исходное положение. Исходя из угла, на который закручивалась нить можно было судить о значении силы тока.

В результате нового эксперимента Ом пришел к формуле:

Х = a / b + l

Здесь X – интенсивность магнитного поля провода, l – длина провода, a – постоянная величина напряжения источника, b – постоянная сопротивления остальных элементов цепи.

Если обратиться к современным терминам для описания данной формулы, то мы получим, что Х – сила тока, а – ЭДС источника, b + l – общее сопротивление цепи .

Закон Ома для участка цепи

Закон Ома для отдельного участка цепи гласит: сила тока на участке цепи увеличивается при возрастании напряжения и уменьшается при возрастании сопротивления этого участка.

I = U / R

Исходя из этой формулы, мы можем решить, что сопротивление проводника зависит от разности потенциалов. С точки зрения математики, это правильно, но ложно с точки зрения физики. Эта формула применима только для расчета сопротивления на отдельном участке цепи.

Таким образом формула для расчета сопротивления проводника примет вид:

R = p ⋅ l / s
Закон Ома для полной цепи

Отличие закона Ома для полной цепи от закона Ома для участка цепи заключается в том, что теперь мы должны учитывать два вида сопротивления. Это «R» сопротивление всех компонентов системы и «r» внутреннее сопротивление источника электродвижущей силы. Формула таким образом приобретает вид:

I = U / R + r
Закон Ома для переменного тока

Переменный ток отличается от постоянного тем, что он изменяется с определенными временными периодами. Конкретно он изменяет свое значение и направление. Чтобы применить закон Ома здесь нужно учитывать, что сопротивление в цепи с постоянным током может отличатся от сопротивления в цепи с током переменным. И отличается оно в том случае если в цепи применены компоненты с реактивным сопротивлением. Реактивное сопротивление может быть индуктивным (катушки, трансформаторы, дроссели) и емкостными (конденсатор).

Попробуем разобраться, в чем реальная разница между реактивным и активным сопротивлением в цепи с переменным током. Вы уже должны были понять, что значение напряжение и силы тока в такой цепи меняется со временем и имеют, грубо говоря, волновую форму.

Если мы схематически представим, как с течением времени меняются эти два значения, у нас получится синусоида. И напряжение, и сила тока от нуля поднимаются до максимального значения, затем, опускаясь, проходят через нулевое значение и достигают максимального отрицательного значения. После этого снова поднимаются через нуль до максимального значения и так далее. Когда говорится, что сила тока или напряжение имеет отрицательное значение, здесь имеется ввиду, что они движутся в обратном направлении.

Весь процесс происходит с определенной периодичностью. Та точка, где значение напряжения или силы тока из минимального значения поднимаясь к максимальному значению проходит через нуль называется фазой.

На самом деле, это только предисловие. Вернемся к реактивному и активному сопротивлению. Отличие в том, что в цепи с активным сопротивлением фаза тока совпадает с фазой напряжения. То есть, и значение силы тока, и значение напряжения достигают максимума в одном направлении одновременно. В таком случае наша формула для расчета напряжения, сопротивления или силы тока не меняется.

Если же цепь содержит реактивное сопротивление, фазы тока и напряжения сдвигаются друг от друга на ¼ периода. Это означает, что, когда сила тока достигнет максимального значения, напряжение будет равняться нулю и наоборот. Когда применяется индуктивное сопротивление, фаза напряжения «обгоняет» фазу тока. Когда применяется емкостное сопротивление, фаза тока «обгоняет» фазу напряжения.

Формула для расчета падения напряжения на индуктивном сопротивлении:

U = I ⋅ ωL

Где L – индуктивность реактивного сопротивления, а ω – угловая частота (производная по времени от фазы колебания).

Формула для расчета падения напряжения на емкостном сопротивлении:

U = I / ω ⋅ С

С – емкость реактивного сопротивления.

Эти две формулы – частные случаи закона Ома для переменных цепей.

Полный же будет выглядеть следующем образом:

I = U / Z

Здесь Z – полное сопротивление переменной цепи известное как импеданс.

Говорят: «не знаешь закон Ома – сиди дома». Так давайте же узнаем (вспомним), что это за закон, и смело пойдем гулять.

Основные понятия закона Ома

Как понять закон Ома? Нужно просто разобраться в том, что есть что в его определении. И начать следует с определения силы тока, напряжения и сопротивления.

Сила тока I

Пусть в каком-то проводнике течет ток. То есть, происходит направленное движение заряженных частиц – допустим, это электроны. Каждый электрон обладает элементарным электрическим зарядом (e= -1,60217662 × 10 -19 Кулона). В таком случае через некоторую поверхность за определенный промежуток времени пройдет конкретный электрический заряд, равный сумме всех зарядов протекших электронов.

Отношение заряда к времени и называется силой тока. Чем больший заряд проходит через проводник за определенное время, тем больше сила тока. Сила тока измеряется в Амперах .

Напряжение U, или разность потенциалов

Это как раз та штука, которая заставляет электроны двигаться. Электрический потенциал характеризует способность поля совершать работу по переносу заряда из одной точки в другую. Так, между двумя точками проводника существует разность потенциалов, и электрическое поле совершает работу по переносу заряда.

Физическая величина, равная работе эффективного электрического поля при переносе электрического заряда, и называется напряжением. Измеряется в Вольтах . Один Вольт – это напряжение, которое при перемещении заряда в 1 Кл совершает работу, равную 1 Джоуль .

Сопротивление R

Ток, как известно, течет в проводнике. Пусть это будет какой-нибудь провод. Двигаясь по проводу под действием поля, электроны сталкиваются с атомами провода, проводник греется, атомы в кристаллической решетке начинают колебаться, создавая электронам еще больше проблем для передвижения. Именно это явление и называется сопротивлением. Оно зависит от температуры, материала, сечения проводника и измеряется в Омах .


Формулировка и объяснение закона Ома

Закон немецкого учителя Георга Ома очень прост. Он гласит:

Сила тока на участке цепи прямо пропорционально напряжению и обратно пропорциональна сопротивлению.

Георг Ом вывел этот закон экспериментально (эмпирически) в 1826 году. Естественно, чем больше сопротивление участка цепи, тем меньше будет сила тока. Соответственно, чем больше напряжение, тем и ток будет больше.

Кстати! Для наших читателей сейчас действует скидка 10% на

Данная формулировка закона Ома – самая простая и подходит для участка цепи. Говоря «участок цепи» мы подразумеваем, что это однородный участок, на котором нет источников тока с ЭДС. Говоря проще, этот участок содержит какое-то сопротивление, но на нем нет батарейки, обеспечивающей сам ток.

Если рассматривать закон Ома для полной цепи, формулировка его будет немного иной.

Пусть у нас есть цепь, в ней есть источник тока, создающий напряжение, и какое-то сопротивление.

Закон запишется в следующем виде:

Объяснение закона Ома для полой цепи принципиально не отличается от объяснения для участка цепи. Как видим, сопротивление складывается из собственно сопротивления и внутреннего сопротивления источника тока, а вместо напряжения в формуле фигурирует электродвижущая сила источника.

Кстати, о том, что такое что такое ЭДС , читайте в нашей отдельной статье.

Как понять закон Ома?

Чтобы интуитивно понять закон Ома, обратимся к аналогии представления тока в виде жидкости. Именно так думал Георг Ом, когда проводил опыты, благодаря которым был открыт закон, названный его именем.

Представим, что ток – это не движение частиц-носителей заряда в проводнике, а движение потока воды в трубе. Сначала воду насосом поднимают на водокачку, а оттуда, под действием потенциальной энергии, она стремиться вниз и течет по трубе. Причем, чем выше насос закачает воду, тем быстрее она потечет в трубе.

Отсюда следует вывод, что скорость потока воды (сила тока в проводе) будет тем больше, чем больше потенциальная энергия воды (разность потенциалов)

Сила тока прямо пропорциональна напряжению.

Теперь обратимся к сопротивлению. Гидравлическое сопротивление – это сопротивление трубы, обусловленное ее диаметром и шероховатостью стенок. Логично предположить, что чем больше диаметр, тем меньше сопротивление трубы, и тем большее количество воды (больший ток) протечет через ее сечение.

Сила тока обратно пропорциональна сопротивлению.

Такую аналогию можно проводить лишь для принципиального понимания закона Ома, так как его первозданный вид – на самом деле довольно грубое приближение, которое, тем не менее, находит отличное применение на практике.

В действительности, сопротивление вещества обусловлено колебанием атомов кристаллической решетки, а ток – движением свободных носителей заряда. В металлах свободными носителями являются электроны, сорвавшиеся с атомных орбит.


В данной статье мы постарались дать простое объяснение закона Ома. Знание этих на первый взгляд простых вещей может сослужить Вам неплохую службу на экзамене. Конечно, мы привели его простейшую формулировку закона Ома и не будем сейчас лезть в дебри высшей физики, разбираясь с активным и реактивным сопротивлениями и прочими тонкостями.

Если у Вас возникнет такая необходимость, Вам с удовольствием помогут сотрудники нашего . А напоследок предлагаем Вам посмотреть интересное видео про закон Ома. Это действительно познавательно!

В 1826 году немецкий ученый Георг Ом совершил открытие и описал
эмпирический закон о соотношении между собой таких показателей как сила тока, напряжение и особенности проводника в цепи. Впоследствии, по имени ученого он стал называться закон Ома.

В дальнейшем выяснилось, что эти особенности не что иное, как сопротивление проводника, возникающее в процессе его контакта с электричеством. Это внешнее сопротивление (R). Есть также внутреннее сопротивление (r), характерное для источника тока.

Закон Ома для участка цепи

Согласно обобщенному закону Ома для некоторого участка цепи, сила тока на участке цепи прямо пропорциональна напряжению на концах участка и обратно пропорциональна сопротивлению.

Где U – напряжение концов участка,I– сила тока, R– сопротивление проводника.

Беря во внимание вышеприведенную формулу, есть возможность найти неизвестные значенияUиR, сделав несложные математические операции.

Данные выше формулы справедливы лишь когда сеть испытывает на себе одно сопротивление.

Закон Ома для замкнутой цепи

Сила тока полной цепи равна ЭДС, деленной на сумму сопротивлений однородного и неоднородного участков цепи.

Замкнутая сеть имеет одновременно сопротивления внутреннего и внешнего характера. Поэтому формулы отношения будут уже другими.

Где E – электродвижущая сила (ЭДС), R- внешнее сопротивление источника, r-внутреннее сопротивление источника.

Закон Ома для неоднородного участка цепи

Замкнутая электрическая сеть содержит участки линейного и нелинейного характера. Участки, не имеющие источника тока и не зависящие от стороннего воздействия являются линейными, а участки, содержащие источник – нелинейными.

Закон Ома для участка сети однородного характера был изложен выше. Закон на нелинейном участке будет иметь следующий вид:

I = U/ R = f1 – f2 + E/ R

Где f1 – f2 – разница потенциалов на конечных точках рассматриваемого участка сети

R – общее сопротивление нелинейного участка цепи

ЭДС нелинейного участка цепи бывает больше нуля или меньше. Если направление движения тока, идущего из источника с движением тока в электрической сети, совпадают, будет преобладать движение зарядов положительного характера и ЭДС будет положительная. В случае же совпадения направлений, в сети будет увеличено движение отрицательных зарядов, создаваемых ЭДС.

Закон Ома для переменного тока

При имеющейся в сети емкости или инертности, необходимо учитывать при проводимых вычислениях, что они выдают свое сопротивление, от действия которого ток приобретает переменный характер.

Закон Ома для переменного тока выглядит так:

где Z – сопротивление по всей длине электрической сети. Его еще называют импеданс. Импеданс составляют сопротивления активного и реактивного характера.

Закон Ома не является основным научным законом, а лишь эмпирическим отношением, причем в некоторых условиях оно может не соблюдаться:

  • Когда сеть обладает высокой частотой, электромагнитное поле меняется с большой скоростью, и при расчетах необходимо учитывать инертность носителей заряда;
  • В условиях низкой температуры с веществами, которые обладают сверхпроводимостью;
  • Когда проводник сильно нагревается проходящим напряжением, отношение тока к напряжению становится переменным и может не соответствовать общему закону;
  • При нахождении под высоким напряжением проводника или диэлектрика;
  • В светодиодных лампах;
  • В полупроводниках и полупроводниковых приборах.

В свою очередь элементы и проводники, соблюдающие закон Ома, называются омическими.

Закон Ома может дать объяснение некоторым явлениям природы. Например, когда мы видим птиц, сидящих на высоковольтных проводах, у нас возникает вопрос – почему на них не действует электрический ток? Объясняется это довольно просто. Птицы, сидя на проводах, представляют собой своеобразные проводники. Большая часть напряжения приходится на промежутки между птицами, а та доля, что приходится на сами «проводники» не представляет для них опасности.

Но это правило работает лишь при единичном соприкосновении. Если птица заденет клювом или крылом провод или телеграфный столб, она неминуемо погибнет от огромного количества напряжения, которое несут в себе эти участки. Такие случаи происходят повсеместно. Поэтому в целях безопасности в некоторых населенных пунктах установлены специальные приспособления, защищающие птиц от опасного напряжения. На таких насестах птицы находятся в полной безопасности.

Закон Ома также широко применятся на практике. Электричество смертельно опасно для человека при одном лишь касании к оголенному проводу. Но в некоторых случаях сопротивление человеческого тела может быть разным.

Так, например, сухая и неповрежденная кожа обладает большим сопротивлением к воздействию электричества нежели рана или кожа, покрытая потом. В следствие переутомления, нервного напряжения и опьянения, даже при небольшом напряжении тока человек может получить сильный удар током.

В среднем, сопротивление тела человека – 700 Ом, значит, для человека является безопасным напряжение в 35 В. Работая с большим напряжением, специалисты используют .

Цепи постоянного тока

— Часть A

Схема

В этом разделе перечислены основные компоненты постоянного тока. цепей (постоянного тока), с которыми вы столкнетесь на уроках физики.

Символ Описание
Это источник ЭДС (электродвижущей силы) с напряжением \ (\ varepsilon \), измеряемым в вольтах, \ (В. \). Наиболее распространенным источником, который вы увидите, будет аккумулятор.
Это сопротивление, измеряемое в единицах Ом Ом, \ (\ Омега \). Чаще всего это будет резистор. Однако провода, соединяющие части цепи, на самом деле не без сопротивления. Мы можем смоделировать этот реальный случай, вставив в цепь «резистор», имеющий такое же сопротивление, как и провода. Таким образом, провода по-прежнему можно считать не имеющими сопротивления, поскольку их сопротивление представлено дополнительным резистором.
Это конденсатор с емкостью \ (C \), измеряемой в фарадах, \ (F.\) Идеальный конденсатор не будет иметь сопротивления (или индуктивности; см. Далее), и мы предположим, что этот символ представляет такой идеальный конденсатор. Если конденсатор не идеален, то есть с ним связано какое-то сопротивление (или индуктивность), то мы можем смоделировать это, поместив «резистор» (или катушку индуктивности) параллельно конденсатору. Этот резистор (или катушка индуктивности) будет иметь такое же сопротивление (или индуктивность), что и неидеальный конденсатор.
Это индуктор с индуктивностью \ (L, \), измеряемой в генри, \ (H.\) Чтобы смоделировать неидеальную катушку индуктивности в цепи (катушка индуктивности с соответствующим сопротивлением или емкостью), мы можем подключить резистор последовательно или конденсатор параллельно катушке индуктивности. Если дать им те же значения, что и у индуктора, это будет представлять собой несовершенный индуктор.

Закон Ома

  1. Закон Ома определяет соотношение между напряжением и током в идеальном проводнике. Это соотношение утверждает, что:
    Разность потенциалов (напряжение) на идеальном проводнике пропорциональна току, проходящему через него.
    Константа пропорциональности называется «сопротивлением», \ (R. \)
    Закон Ома определяется по формуле:
    \ (V = IR \)
    где \ (V \) — разность потенциалов между двумя точками, которые включают сопротивление \ (R. \) \ (I \) — это ток, протекающий через сопротивление. Для биологической работы часто предпочтительнее использовать проводимость \ (g = 1 / R \); В этой форме закон Ома:
    \ (I = g V \)
  2. Материал, который подчиняется закону Ома, называется «омический» или «линейный» , потому что разность потенциалов на нем изменяется линейно с током.
  3. Закон
  4. Ома можно использовать для решения простых схем. Полная схема — это такая, которая является замкнутым контуром. Он содержит по крайней мере один источник напряжения (что обеспечивает увеличение потенциальной энергии) и по крайней мере одно падение потенциала, то есть место, где потенциальная энергия уменьшается. Сумма напряжений во всей цепи равна нулю.
  5. Увеличение потенциальной энергии в цепи заставляет заряд переходить от более низкого к более высокому потенциалу (т. Е. Напряжению). Обратите внимание на разницу между потенциальной энергией и потенциалом.

    Из-за электростатической силы, которая пытается переместить положительный заряд от более высокого к более низкому потенциалу, должна быть другая «сила» для перемещения заряда от более низкого потенциала к более высокому внутри батареи. Эта так называемая сила называется электродвижущей силой или ЭДС. Единицей измерения ЭДС в системе СИ является вольт (и, таким образом, это не совсем сила, несмотря на ее название). Мы будем использовать скрипт E, символ \ (\ varepsilon \), для представления ЭДС.

    Уменьшение потенциальной энергии может происходить различными способами.Например, потеря тепла в цепи из-за некоторого электрического сопротивления может быть одним из источников потери энергии.

    Поскольку энергия сохраняется, разность потенциалов на ЭДС должна быть равна разности потенциалов на остальной части цепи. То есть будет выполняться закон Ома:
    \ (\ varepsilon = I R \)

Резисторы серии

резисторы можно подключать последовательно; то есть ток течет через них один за другим. Схема на Рисунке 1 показывает три резистора, включенных последовательно, а направление тока указано стрелкой.

Рисунок 1: Резисторы, подключенные последовательно

Обратите внимание, что, поскольку существует только один путь для прохождения тока, ток через каждый из резисторов одинаков.

  1. \ (I = I_1 = I_2 = I_3 \)
    Кроме того, падение напряжения на резисторах должно составлять общее напряжение, подаваемое батареей:
  2. \ (V_ \ mathrm {total} = V_1 + V_2 + V_3 \)
    Поскольку \ (V = I R \), то
  3. \ (V_ \ mathrm {total} = I_1 R_1 + I_2R_2 + I_3R_3 \)
    Но закон Ома также должен выполняться для всей цепи:
  4. \ (V_ \ mathrm {total} = I \ cdot R_ \ mathrm {equal} \)
    Установка уравнений 3.и 4. равно, получаем:
  5. \ (I \ cdot R_ \ mathrm {эквивалент} = I_1 \ cdot R_1 + I_2 \ cdot R_2 + I_3 \ cdot R_3 \)
    Мы знаем, какой ток через каждый резистор (из уравнения 1.) равен \ (I \) .
  6. \ (I \ cdot R _ \ mathrm {эквивалент} = I (R_1 + R_2 + R_3) \)
    Таким образом, токи компенсируются с обеих сторон, и мы приходим к выражению для эквивалентного сопротивления для резисторов, соединенных последовательно.
  7. \ (R_ \ mathrm {эквивалент} = R_1 + R_2 + R_3 \)
    Как правило, эквивалентное сопротивление резисторов, соединенных последовательно, является суммой их сопротивлений.То есть
  8. \ (R_ \ mathrm {эквивалент} = \ sum R_i \)
    Это также можно записать в терминах проводимости, поскольку проводимость просто обратна сопротивлению:
  9. \ (\ frac {1} {g} = \ sum \ frac {1} {g_1} \)

Параллельные резисторы

Резисторы

могут быть подключены таким образом, что они ответвляются из одной точки (известной как узел) и снова соединяются где-то еще в цепи. Это называется параллельным подключением. Каждый из трех резисторов на рисунке 1 — это еще один путь для прохождения тока между точками \ (A \) и \ (B.\)

Рисунок 2: Пример схемы, содержащей три резистора, включенных параллельно

Рисунок 3: Схема, содержащая резисторы, включенные параллельно, эквивалентна Рисунку 2


Обратите внимание, что узел не обязательно должен быть физически одной точкой; пока у тока есть несколько альтернативных путей, эта часть цепи считается параллельной. На рисунках 1 и 2 показаны идентичные схемы, но с разным внешним видом.

При \ (A \) потенциал должен быть одинаковым для каждого резистора.Точно так же на \ (B \) потенциал также должен быть одинаковым для каждого резистора. Итак, между точками \ (A \) и \ (B, \) разность потенциалов одинакова. То есть каждый из трех резисторов в параллельной цепи должен иметь одинаковое напряжение.

  1. \ (V_1 = V_2 = V_3 = V \)
    Кроме того, ток разделяется по мере продвижения от \ (A \) к \ (B. \). Итак, сумма токов через три ветви — это такой же, как ток на \ (A \) и на \ (B \) (где токи от ответвления объединяются).
  2. \ (I = I_1 + I_2 + I_3 \)
    По закону Ома уравнение 2 эквивалентно:
  3. \ (\ frac {V} {R_ \ mathrm {equal}} = \ frac {V_1} {R_1} + \ frac {V_2} {R_2} + \ frac {V_3} {R_3} \)
    По уравнению 1. , мы видим, что все напряжения равны. Итак, V отменяется, и мы остаемся с
  4. \ (\ frac {1} {R_ \ mathrm {equal}} = \ frac {1} {R_1} + \ frac {1} {R_2} + \ frac {1} {R_3} \)
    Этот результат может быть обобщены на любое количество параллельно включенных резисторов.
  5. \ (\ frac {1} {R_ \ mathrm {equal}} = \ sum \ frac {1} {R_i} \)
    Поскольку сопротивление является обратной величиной проводимости, уравнение [5] может быть выражено в терминах проводимости.
  6. \ (g_ \ mathrm {эквивалент} = \ сумма g_i \)

4.11 Цепи постоянного тока, содержащие резисторы и конденсаторы — Физика Дугласского колледжа 1207

Резюме

  • Объясните важность постоянной времени τ и вычислите постоянную времени для заданного сопротивления и емкости.
  • Объясните, почему батарейки в фонарике постепенно разряжаются, а свет со временем тускнеет.
  • Опишите, что происходит с графиком зависимости напряжения на конденсаторе от времени при его зарядке.
  • Объясните, как работает схема синхронизации, и перечислите некоторые приложения.
  • Рассчитайте необходимую скорость стробоскопической вспышки, необходимую для «остановки» движения объекта на определенной длине.

При использовании камеры со вспышкой зарядка конденсатора, питающего вспышку, занимает несколько секунд.Световая вспышка разряжает конденсатор за крошечные доли секунды. Почему зарядка занимает больше времени, чем разрядка? Этот вопрос и ряд других явлений, связанных с зарядкой и разрядкой конденсаторов, обсуждаются в этом модуле.

Цепь RC — это цепь, содержащая резистор R и конденсатор C . Конденсатор — это электрический компонент, в котором хранится электрический заряд.

На рисунке 1 показана простая схема RC , в которой используется источник постоянного напряжения.Конденсатор изначально не заряжен. Как только переключатель замыкается, ток течет к первоначально незаряженному конденсатору и от него. По мере увеличения заряда на пластинах конденсатора сопротивление потоку заряда усиливается за счет отталкивания одинаковых зарядов на каждой пластине.

Что касается напряжения, это связано с тем, что напряжение на конденсаторе равно В c = Q / C , где Q — это количество заряда, накопленного на каждой пластине, а C — это емкость .Это напряжение противодействует батарее, возрастая от нуля до максимальной ЭДС при полной зарядке. Таким образом, ток уменьшается от своего начального значения до нуля, когда напряжение на конденсаторе достигает того же значения, что и ЭДС. При отсутствии тока отсутствует падение IR , поэтому напряжение на конденсаторе должно быть равно ЭДС источника напряжения. Это также можно объяснить вторым правилом Кирхгофа (правилом цикла), обсуждаемым в Правилах Кирхгофа, согласно которому алгебраическая сумма изменений потенциала вокруг любого замкнутого контура должна быть равна нулю.

Начальный ток равен I 0 = ЭДС / R , потому что все падение IR находится в сопротивлении. Следовательно, чем меньше сопротивление, тем быстрее будет заряжаться данный конденсатор. Обратите внимание, что внутреннее сопротивление источника напряжения включено в R , как и сопротивление конденсатора и соединительных проводов. В приведенном выше сценарии камеры со вспышкой, когда батареи, питающие камеру, начинают изнашиваться, их внутреннее сопротивление возрастает, уменьшая ток и увеличивая время, необходимое для подготовки к следующей вспышке.

Рис. 1. (a) Схема RC с изначально незаряженным конденсатором. Как только переключатель замкнут, ток течет в указанном направлении (противоположном потоку электронов). Взаимное отталкивание одинаковых зарядов в конденсаторе постепенно замедляет поток по мере заряда конденсатора, прекращая ток, когда конденсатор полностью заряжен и Q = C ЭДС . (Помните, что Q = CV) (б) График зависимости напряжения на конденсаторе от времени, когда переключатель замыкается в момент времени t = 0 .(Обратите внимание, что в двух частях рисунка заглавная буква E обозначает ЭДС, q обозначает заряд, накопленный на конденсаторе, а τ — постоянная времени RC .)

Напряжение. на конденсаторе изначально равен нулю и сначала быстро нарастает, так как начальный ток максимален. На рисунке 1 (b) показан график зависимости напряжения конденсатора от времени (t ), начиная с момента включения переключателя при t = 0 . Напряжение асимптотически приближается к ЭДС, поскольку чем ближе оно к ЭДС, тем меньше протекает ток.Уравнение для зависимости напряжения от времени при зарядке конденсатора C через резистор R , полученное с помощью расчетов, равно

, где В — напряжение на конденсаторе, ЭДС равна ЭДС источника постоянного напряжения, а экспонента e = 2,718… является основанием натурального логарифма. Обратите внимание, что единицы измерения RC — секунды. Определяем

τ = RC

, где τ (греческая буква тау) называется постоянной времени для цепи RC .Как отмечалось ранее, небольшое сопротивление R позволяет конденсатору заряжаться быстрее. Это разумно, поскольку больший ток протекает через меньшее сопротивление. Также разумно, что чем меньше конденсатор C , тем меньше времени потребуется для его зарядки. Оба множителя содержатся в τ = RC.

Более количественно, рассмотрим, что происходит, когда t = τ = RC. Тогда напряжение на конденсаторе

Это означает, что за время τ = RC напряжение возрастает до 0.632 от его окончательного значения. Напряжение повысится на 0,632 от остатка в в следующем раз в τ = RC . Характерной чертой экспоненциальной функции является то, что конечное значение никогда не достигается, но 0,632 остатка от этого значения достигается каждый раз, τ. Таким образом, всего за несколько кратных постоянной времени τ , конечное значение почти достигнуто, как показано на графике на Рисунке 1 (b).

Разряд конденсатора через резистор происходит аналогично, как показано на рисунке 2.Первоначально ток равен I o = V o / R , управляемый начальным напряжением V o на конденсаторе. По мере уменьшения напряжения ток и, следовательно, скорость разряда уменьшается, что подразумевает другую экспоненциальную формулу для В . С помощью расчетов напряжение В на конденсаторе C , разряженном через резистор R , оказывается равным

. Рисунок 2. (a) При замыкании переключателя конденсатор C разряжается через резистор R .Взаимное отталкивание одинаковых зарядов на каждой пластине приводит в движение ток. (б) График зависимости напряжения на конденсаторе от времени: В = 0 при t = 0 . Напряжение уменьшается экспоненциально, падая на фиксированном отрезке пути до нуля в каждой последующей постоянной времени τ .

График на Рисунке 2 (b) является примером этого экспоненциального затухания. Опять же, постоянная времени равна τ = RC . Небольшое сопротивление R позволяет конденсатору разряжаться за малое время, так как ток больше.Точно так же небольшая емкость требует меньше времени для разряда, поскольку сохраняется меньше заряда. В первом временном интервале τ = RC . после замыкания переключателя напряжение падает до 0,368 от своего начального значения, так как В = Vo e -1 = 0,368 В o .

В течение каждого последующего времени τ напряжение падает до 0,368 от своего предыдущего значения. Через несколько значений, кратных τ , напряжение становится очень близким к нулю, как показано на графике на Рисунке 2 (b).

Теперь мы можем объяснить, почему вспышка камеры в нашем сценарии заряжается намного дольше, чем разряжается; сопротивление при зарядке значительно больше, чем при разрядке. Внутреннее сопротивление батареи составляет большую часть сопротивления во время зарядки. По мере старения аккумулятора возрастающее внутреннее сопротивление делает процесс зарядки еще медленнее. (Вы могли это заметить.)

Импульсный разряд происходит через ионизованный газ с низким сопротивлением в импульсной трубке и происходит очень быстро.Фотографии со вспышкой, такие как на рисунке 3, могут запечатлеть краткий момент быстрого движения, потому что вспышка может иметь длительность менее микросекунды. Такие вспышки могут быть очень интенсивными.

Во время Второй мировой войны ночные разведывательные фотографии производились с воздуха, при этом одна вспышка освещала территорию противника более чем на квадратный километр. Краткость вспышки устраняет размытость изображения из-за движения самолета наблюдения. Сегодня очень важно использовать мощные импульсные лампы для накачки энергии в лазер.Короткая интенсивная вспышка может быстро возбудить лазер и позволить ему переизлучить энергию в другой форме.

Рисунок 3. Эта покадровая фотография рыжего колибри ( Selasphorus rufus ), питающегося цветком, была получена с помощью чрезвычайно короткой и интенсивной вспышки света, вызванной разрядом конденсатора через газ. (Источник: Дин Э. Биггинс, Служба рыболовства и дикой природы США)

Комплексная концептуальная проблема: расчет размера конденсатора — стробоскопы

Фотографию со скоростной вспышкой впервые применил Док Эдгертон в 1930-х годах, когда он был профессором электротехники в Массачусетском технологическом институте.Вы, возможно, видели примеры его работ на удивительных снимках движущихся колибри, капли молока, брызгающей на стол, или пули, пробивавшей яблоко (см. Рис. 3). Как упоминалось ранее в этом модуле, чтобы остановить движение и запечатлеть эти изображения, нужна очень короткая импульсная вспышка высокой интенсивности.

Предположим, кто-то хочет сделать снимок пули (движущейся со скоростью 500 м / с) , проходящей через яблоко. Продолжительность вспышки связана с постоянной времени RC , τ = RC .Конденсатор какого размера потребовался бы в схеме RC для успеха, если бы сопротивление импульсной лампы было 10,0 Ом? Предположим, что яблоко представляет собой сферу диаметром 8,0 x10 -2 м .

Стратегия

Начнем с определения задействованных физических принципов. В этом примере рассматривается стробоскоп, о котором говорилось выше. На рисунке 2 показана схема этого пробника. Характерное время строба τ задается как τ = RC .

Решение

Мы хотим найти C , но мы не знаем τ = RC .. Мы хотим, чтобы вспышка была включена только тогда, когда пуля пересекает яблоко. Поэтому нам нужно использовать кинематические уравнения, которые описывают взаимосвязь между расстоянием x , постоянной скоростью v и временем t :

x = vt или t = x / v

Скорость пули равна 5,0 x 10 2 м / с , а расстояние x равно 8.0 x 10 -2 м . Таким образом, время перехода составляет

.

t = x / v = (8,0 x 10 -2 м) / (5,0 x 10 2 м / с) = 1,6 x 10 -4 с

Мы устанавливаем это значение для времени пересечения t равным τ . Следовательно,

C = τ / R = t / R = (1,6 x 10 -4 с) (10,0 Ом) = 16 мкФ.

(Примечание: емкость C обычно измеряется в фарадах, F , определяется как кулоны на вольт.Из уравнения видно, что C также может быть выражено в секундах на Ом.)

Обсуждение

Интервал вспышки 160 мкс (время перемещения пули) сегодня относительно легко получить. Стробоскопы открыли новые миры от науки до развлечений. Информация с изображения яблока и пули была использована в отчете комиссии Уоррена об убийстве президента Джона Ф. Кеннеди в 1963 году, чтобы подтвердить, что была выпущена только одна пуля.

Цепи RC обычно используются для синхронизации. Банальный пример этого можно найти в повсеместных прерывистых системах стеклоочистителей современных автомобилей. Время между протиранием можно изменять, регулируя сопротивление в цепи RC . Другой пример схемы RC можно найти в новых украшениях, костюмах на Хэллоуин и различных игрушках, которые имеют мигающие огни с батарейным питанием. (Схема синхронизации см. На Рисунке 4.)

Более важное применение схем RC для определения времени — это искусственный кардиостимулятор, используемый для контроля частоты сердечных сокращений.Частота сердечных сокращений обычно контролируется электрическими сигналами, генерируемыми сино-предсердным (SA) узлом, который находится на стенке камеры правого предсердия. Это заставляет мышцы сокращаться и перекачивать кровь. Иногда сердечный ритм ненормальный, а сердцебиение слишком высокое или слишком низкое.

Искусственный кардиостимулятор устанавливается рядом с сердцем, чтобы при необходимости передавать в сердце электрические сигналы с соответствующей постоянной времени. У кардиостимуляторов есть датчики, которые обнаруживают движение тела и дыхание, чтобы увеличить частоту сердечных сокращений во время упражнений, чтобы удовлетворить повышенные потребности организма в крови и кислороде.

Рис. 4. (a) Лампа в этой цепи RC обычно имеет очень высокое сопротивление, так что батарея заряжает конденсатор, как если бы лампы там не было. Когда напряжение достигает порогового значения, через лампу протекает ток, который резко снижает ее сопротивление, и конденсатор разряжается через лампу, как если бы батареи и зарядного резистора не было. После разряда процесс начинается снова с периодом мигания, определяемым константой RC τ .(б) График зависимости напряжения от времени для этой цепи.

Время расчета:

RC Цепь в дефибрилляторе сердца

Дефибриллятор сердца используется для реанимации жертвы аварии путем разряда конденсатора через туловище ее тела. Упрощенная версия схемы показана на рисунке 2. (a) Какова постоянная времени, если используется конденсатор 8,00 — мкФ , а сопротивление пути через его тело составляет 1,00 x 10 3 Ом ? (b) Если начальное напряжение равно 10.0 кВ, сколько времени нужно, чтобы снизиться до 5,00 x 10 2 В ?

Стратегия

Так как сопротивление и емкость даны, их просто умножить, чтобы получить постоянную времени, запрашиваемую в части (а). Чтобы найти время, за которое напряжение снизится до 5,00 x 10 2 В , мы многократно умножаем начальное напряжение на 0,368 до тех пор, пока не будет получено напряжение, меньшее или равное 5,00 x 10 2 В .Каждое умножение соответствует времени τ секунд.

Решение для (а)

Постоянная времени τ определяется уравнением RC . Ввод заданных значений сопротивления и емкости (и помня, что единицы для фарада могут быть выражены как с / Ом дает

τ = RC = (1,00 x10 3 Ом) (8,00 мкФ) = 8,00 мс

Решение для (b)

В первых 8.00 мс, напряжение (10,0 кВ) снижается до 0,368 от исходного значения. То есть:

В = 0,368 В o = 3,680 x 10 3 В при t = 8,00 мс.

(Обратите внимание, что мы переносим дополнительную цифру для каждого промежуточного вычисления.) Еще через 8,00 мс мы снова умножаем на 0,368, и напряжение равно

Аналогично, еще через 8,00 мс напряжение

Обсуждение

Таким образом, всего через 24,0 мс напряжение упало до 498 В, или 4.98% от первоначальной стоимости. Такие короткие промежутки времени полезны при дефибрилляции сердца, потому что кратковременный, но интенсивный ток вызывает кратковременное, но эффективное сокращение сердца. Фактическая схема в дефибрилляторе сердца немного сложнее, чем на рисунке 2, чтобы компенсировать магнитные эффекты и эффекты переменного тока, которые будут рассмотрены в разделе «Магнетизм».

Проверьте свое понимание

1: Когда разность потенциалов на конденсаторе равна ЭДС?

Исследование PhET: комплект для конструирования цепей (только для постоянного тока)

Комплект электроники в вашем компьютере.Создавайте схемы с резисторами, лампочками, батареями и переключателями.

Прямая ссылка: https://phet.colorado.edu/sims/html/capacitor-lab-basics/latest/capacitor-lab-basics_en.html

Измеряйте реалистичные амперметр и вольтметр. Просмотрите схему как схематическую диаграмму или переключитесь в реалистичный вид. Прямая ссылка: https://phet.colorado.edu/sims/html/circuit-construction-kit-dc/latest/circuit-construction-kit-dc_en.html

Рисунок 5.Комплект для конструирования цепей (только для постоянного тока)

Концептуальные вопросы

1: Что касается единиц, участвующих в соотношении τ = RC , убедитесь, что единицы измерения сопротивления, умноженные на емкость, равны времени, то есть Ом • F = с.

2: Постоянная времени RC при дефибрилляции сердца имеет решающее значение для ограничения времени протекания тока. Если емкость в дефибрилляционном блоке фиксированная, как бы вы управляли сопротивлением в цепи, чтобы отрегулировать постоянную RC τ ? Потребуется ли также регулировка приложенного напряжения, чтобы обеспечить надлежащее значение подаваемого тока?

3: При измерении ЭКГ важно измерять колебания напряжения за небольшие промежутки времени.Время ограничено константой схемы RC — невозможно измерить изменения времени короче, чем RC . Как бы вы управляли R и C в цепи, чтобы обеспечить необходимые измерения?

4: Нарисуйте два графика зависимости заряда конденсатора от времени. Нарисуйте один для зарядки первоначально незаряженного конденсатора последовательно с резистором, как в схеме на рисунке 1, начиная с t = 0 секунд . Нарисуйте другой для разряда конденсатора через резистор, как в схеме на Рисунке 2, начиная с t = 0 , с начальным зарядом Q o .Покажите хотя бы два интервала τ .

5: При зарядке конденсатора, как описано в связи с рисунком 1, сколько времени требуется, чтобы напряжение на конденсаторе достигло ЭДС? Это проблема?

6: При разрядке конденсатора, как описано в связи с рисунком 2, сколько времени требуется, чтобы напряжение на конденсаторе достигло нуля? Это проблема?

7: Ссылаясь на рисунок 1, нарисуйте график разности потенциалов на резисторе в зависимости от времени, показывающий, по крайней мере, два интервала τ .Также нарисуйте график зависимости тока от времени для этой ситуации.

8: Длинный недорогой удлинитель подключается изнутри дома к холодильнику снаружи. Холодильник не работает должным образом. В чем может быть проблема?

9: Указывает ли график на рисунке 4, что постоянная времени для разрядки меньше, чем для зарядки? Ожидаете ли вы, что ионизированный газ будет иметь низкое сопротивление? Как бы вы отрегулировали R , чтобы увеличить время между вспышками? Повлияет ли регулировка R на время разряда?

10: Электронное устройство может иметь конденсаторы большой емкости с высоким напряжением в секции источника питания, представляющие опасность поражения электрическим током, даже когда устройство выключено.Таким образом, поперек такого конденсатора помещается «спускной резистор», как схематично показано на рисунке 6, для отвода заряда после выключения устройства. Почему сопротивление кровотока должно быть намного больше, чем эффективное сопротивление остальной цепи? Как это влияет на постоянную времени разряда конденсатора?

Рисунок 6. Перепускной резистор R bl разряжает конденсатор в этом электронном устройстве после его выключения.

Проблемные упражнения

1: Устройство синхронизации в системе стеклоочистителей прерывистого действия автомобиля основано на постоянной времени RC и использует конденсатор 0,500 мкФ и переменный резистор. В каком диапазоне R должно изменяться для достижения постоянной времени от 2,00 до 15,0 с?

2: Кардиостимулятор срабатывает 72 раза в минуту, каждый раз, когда конденсатор емкостью 25,0 нФ заряжается (батареей, включенной последовательно с резистором) до 0.632 от его полного напряжения. В чем ценность сопротивления?

3: Продолжительность фотографической вспышки связана с постоянной времени RC , которая для определенной камеры составляет 0,100 мкс . (а) Если сопротивление импульсной лампы составляет 0,0400 Ом во время разряда, каков размер конденсатора, обеспечивающего ее энергию? (b) Какова постоянная времени для зарядки конденсатора, если зарядное сопротивление 800 кОм ?

4: A 2.00 мкФ и конденсатор 7,50 мкФ могут быть подключены последовательно или параллельно, как и резисторы 25,0 кОм и 100 кОм. Вычислите четыре постоянные времени RC , которые можно получить при последовательном соединении полученной емкости и сопротивления.

5: После двух постоянных времени, какой процент конечного напряжения, ЭДС, находится на первоначально незаряженном конденсаторе C , заряженном через сопротивление R ?

6: Резистор 500 Ом, незаряженный 1.Конденсатор 50 мкФ и эдс 6,16 В. включены последовательно. а) Каков начальный ток? (b) Какова постоянная времени RC ? (c) Каков ток через одну постоянную времени? (d) Какое будет напряжение на конденсаторе после одной постоянной времени?

7: Дефибриллятор сердца, используемый на пациенте, имеет постоянную времени RC 10,0 мс из-за сопротивления пациента и емкости дефибриллятора. (a) Если дефибриллятор имеет 8.Емкость 00 мкФ , какое сопротивление пути через пациента? (Вы можете пренебречь емкостью пациента и сопротивлением дефибриллятора.) (B) Если начальное напряжение составляет 12,0 кВ, сколько времени потребуется, чтобы упасть до 6,00 x 10 2 В?

8: Монитор ЭКГ должен иметь постоянную времени RC менее 1,00 x 10 2 мкс, чтобы иметь возможность измерять изменения напряжения через небольшие промежутки времени. (а) Если сопротивление цепи (в основном из-за сопротивления груди пациента) равно 1.00 кОм, какова максимальная емкость цепи? (б) Будет ли сложно на практике ограничить емкость до значения, меньшего, чем значение, указанное в (а)?

9: На рис. 7 показано, как истекающий резистор используется для разряда конденсатора после выключения электронного устройства, что позволяет человеку работать с электроникой с меньшим риском поражения электрическим током. а) Что такое постоянная времени? (b) Сколько времени потребуется, чтобы снизить напряжение на конденсаторе до 0,250% (5% от 5%) от его полного значения после начала разряда? (c) Если конденсатор заряжен до напряжения V o через сопротивление 100 Ом , рассчитайте время, необходимое для повышения до 0.865 В (Это примерно две постоянные времени.)

Рисунок 7.

10: Используя точную экспоненциальную обработку, найдите, сколько времени требуется для разряда конденсатора 250 — через резистор 500 Ом до 1,00% от его первоначального напряжения.

11: Используя точную экспоненциальную обработку, найдите, сколько времени требуется для зарядки первоначально незаряженного конденсатора 100 пФ через резистор 75,0 МОм до 90.0% от его конечного напряжения.

12: интегрированные концепции

Если вы хотите сфотографировать пулю, движущуюся со скоростью 500 м / с, то очень короткая вспышка света, производимая разрядом RC через импульсную трубку, может ограничить размытие. Предполагая, что 1,00 мм движения за одну постоянную RC приемлемо, и учитывая, что вспышка приводится в действие конденсатором 600 мкФ , каково сопротивление в импульсной лампе?

13: интегрированные концепции

Мигающая лампа в рождественской серьге основана на разряде конденсатора RC через его сопротивление.Эффективная продолжительность вспышки составляет 0,250 с, в течение которых она дает в среднем 0,500 Вт при среднем 3,00 В. а) Какую энергию она рассеивает? б) Сколько заряда проходит через лампу? (c) Найдите емкость. (г) Какое сопротивление лампы?

14: интегрированные концепции

Конденсатор емкостью 160 мкФ, заряженный до 450 В, разряжается через резистор 31,2 кОм. (а) Найдите постоянную времени. (b) Рассчитайте повышение температуры резистора, учитывая, что его масса равна 2.50 г, а его удельная теплоемкость составляет 1,67 кДж / кг o ° C, с учетом того, что большая часть тепловой энергии сохраняется за короткое время разряда. (c) Рассчитайте новое сопротивление, предполагая, что это чистый углерод. (d) Кажется ли это изменение сопротивления значительным?

16: Необоснованные результаты

(a) Рассчитайте емкость, необходимую для получения постоянной времени RC 1,00 x 10 3 с с резистором 0,100 Ом. б) Что неразумного в этом результате? (c) Какие допущения ответственны?

17: Постройте свою проблему

Рассмотрим вспышку фотоаппарата.Постройте задачу, в которой вы вычисляете размер конденсатора, который накапливает энергию для лампы-вспышки. Среди факторов, которые необходимо учитывать, — это напряжение, приложенное к конденсатору, энергия, необходимая для вспышки, и соответствующий заряд, необходимый для конденсатора, сопротивление импульсной лампы во время разряда и желаемая постоянная времени RC .

18: Постройте свою проблему

Рассмотрим перезаряжаемый литиевый элемент, который будет использоваться для питания видеокамеры.Постройте задачу, в которой вы вычисляете внутреннее сопротивление ячейки во время нормальной работы. Кроме того, рассчитайте минимальное выходное напряжение зарядного устройства, которое будет использоваться для зарядки литиевого элемента. Среди факторов, которые следует учитывать, — ЭДС и полезное напряжение на клеммах литиевого элемента, а также ток, который он должен обеспечивать в видеокамере.

Глоссарий

RC-цепь
схема, содержащая как резистор, так и конденсатор
конденсатор
Электрический компонент, используемый для хранения энергии путем разделения электрического заряда на двух противоположных пластинах
емкость
максимальное количество электрической потенциальной энергии, которое может быть сохранено (или отделено) для данного электрического потенциала

Решения

Проверьте свое понимание

1: Только когда ток, потребляемый или вводимый в конденсатор, равен нулю.Конденсаторы, как и батареи, имеют внутреннее сопротивление, поэтому их выходное напряжение не является ЭДС, если ток не равен нулю. На практике это сложно измерить, поэтому мы ссылаемся на напряжение конденсатора, а не на его ЭДС. Но источник разности потенциалов в конденсаторе является фундаментальным, и это ЭДС.

Проблемные упражнения

1: от 4,00 до 30,0 МОм

3: (а) 2,50 мкФ (б) 2,00 с

5: 86,5%

7: (а) 1.25 кОм (б) 30,0 мс

9: (а) 20,0 с (б) 120 с (в) 16,0 мс

11: 1,73 x 10 -2 с

12: 3,33 x 10 -3 Ом

15: (а) 4,99 с (б) 3,87 o C (в) 31,1 кОм (г) Нет

Закон

Ома для замкнутой цепи, соотношение между ЭДС (VB) электрического элемента и напряжением на его полюсах

Закон Ома

ЭДС электрического элемента (батареи — источника) — это общая работа, выполняемая внутри и снаружи элемента по передаче электрических зарядов в электрической цепи. Если мы обозначим ЭДС батареи через (V B ), общая сила тока в цепи (I), внешнее сопротивление (R) и внутреннее сопротивление ячейки (r).

Тогда: V B = I R + I r

В В = I (R + r)

I = V B / (R + r)

Это известно как закон Ома для замкнутой цепи, где:

Сила электрического тока = Общая электродвижущая сила / Общее сопротивление цепи

Закон Ома

Отношение между ЭДС (В B ) электрического элемента и напряжением на его полюсах (В)

По закону Ома для замкнутой цепи:

В В = I R + I r, V = I R

V B = V + I r, ∴ V = V B — I r

Из предыдущего соотношения мы видим, что при с внешнее сопротивление (R) увеличивается, электрический ток (I), проходящий в цепи, постепенно уменьшается, а разность потенциалов (V) между полюсами ячейки увеличивается.

Разность потенциалов (V) между полюсами ячейки становится равной ЭДС источника (V B ). Когда значение тока становится очень маленьким, (I r) можно не учитывать.

ЭДС электрического элемента больше, чем разность потенциалов между выводами его внешней цепи, когда цепь включена, потому что внутреннее сопротивление электрического элемента потребляет работу, чтобы пропустить ток внутри электрического элемента на основе соотношения (V B = V + I r) и, следовательно, (V B ).

Следовательно, мы можем определить ЭДС ячейки как:

ЭДС ячейки (V B ) — это разность потенциалов на полюсах ячейки в случае отсутствия тока в цепи (выключатель разомкнут), или это общая работа, проделанная внутри и снаружи ячейки для передачи электрический заряд 1 Кл (единица электрических зарядов) в электрической цепи, ЭДС источника измеряется в вольтах.

Когда ЭДС электрического элемента = 3 В, общая работа, проделанная внутри и снаружи элемента для передачи электрического заряда 1 Кл в электрической цепи = 3 Дж.

Закон Ома

В случае включения в цепь одного электрического элемента:

Где V B — показания вольтметра на батарее, имеющей внутреннее сопротивление r, батарея подключена последовательно с сопротивлением, которое имеет разность потенциалов V 2 , и она подключена последовательно с амперметр.

Если переключатель K замкнут:

I = V B / (R + r)

I = V B — V 1 / r

I = V 2 / R

V 2 = I R, V 1 = V B — I r

Если выключатель К разомкнут:

I = 0

В 2 = 0, В 1 = В В

В случае двух последовательно включенных в цепь электрических ячеек

Где, V 1 — это показание вольтметра на первой батарее с внутренним сопротивлением r 1 , V 2 — это показание вольтметра на второй батарее, внутреннее сопротивление которой r 2 , разность потенциалов на двух батареях составляет V 3 .

Когда две батареи подключены в одном направлении:

I = [(V B ) 1 + (V B ) 2 ] / (R + r 1 + r 2 )

V 1 = (V B ) 1 — I r 1

V 2 = (V B ) 2 — I r 2

В 3 = В 1 + В 2

Когда две батареи подключены в противоположных направлениях, Где (V B ) 2 <(V B ) 1 :

I = [(V B ) 1 — (V B ) 2 ] / (R + r 1 + r 2 )

V 1 = (V B ) 1 — I r 1 (напорный корпус)

В 2 = (В B ) 2 + I r 2 (зарядный кейс)

В 3 = В 1 В 2

Электрический ток, разность потенциалов, электрическое сопротивление и закон Ома

Соединение сопротивлений (последовательно и параллельно), Электроэнергия и Электроэнергия

Первый закон Кирхгофа, второй закон Кирхгофа и способы решения проблем, связанных с законами Кирхгофа

РЕШЕНО: На рисунке показана схема, содержащая электродвижущую силу, конденсатор емкостью C фарад (F) и резистор с сопротивлением R Ом (\ Omega).Падение напряжения на нем равно Q / C, где Q — это заряд (в кулонах), поэтому в этом случае закон Кирхгофа дает RI + \ frac {Q} {C} = E (t), но I = dQ / dt (см. Примеры 3.7.3), поэтому мы имеем R \ frac {dQ} {dt} + \ frac {1} {C} Q = E (t) Предположим, что сопротивление равно 5 \ Омега, емкость — 0,05 Ф, батарея дает постоянное напряжение 60 В, а начальный заряд Q (0) = 0 C. Найдите заряд и ток в момент времени t.

Стенограмма видео

Хорошо, нам сообщили дату возникновения проблемы.R D q. D, T плюс один больше. См. Time’s Q. Equal. К слову о зубах, которые нам также дали, воспользуемся этим позже. Куб из нуля не равен снова. Наше сопротивление. Deke Udt — это ток, который измеряется емкостью при зарядке, и вы дразните подачу напряжения на батарею. Нам дали номера для публикации, так что давайте подключим их. Спасибо, E t. Умножить на пять Так как я был сопротивлением плюс один в 10,5 раза, Q равняется 60. И мы знаем, что это проблема дифференциальных уравнений, и что это не так, потому что мы собираемся использовать метод интегрирования коэффициентов, который мы хотели Простите, постоянный, самый игривый придурок.Ты не будешь одним из них. Итак, у нас есть два перебежчика на пять. Но прежде чем мы это сделаем, было бы легче сделать это, если бы мы просто запомнили этот пункт над точкой. 05 равно 20. Так что позвольте мне это переписать. Итак, пять q d T плюс 20 умножить на мило равняются 60, а когда мы разделим на пять, мы получим d q T T плюс четыре умножения на Q равняется поездке. И поскольку мы используем метод интегрирования факторов, я собираюсь называть его I I чая B к интегралу всего, что умножается. В. Что в данном случае для DT, а интеграл для чая равен всего 40.И поскольку это метод интеграции факторов, о котором нам не о чем беспокоиться, плюс C здесь. Хорошо, теперь, когда у нас есть это, мы можем умножить обе части дифференциального уравнения на 14, что я сделаю в этом информационном бюллетене. Итак, у нас есть 40 умноженных на D. Q. D T плюс четыре потребности, чтобы 40 умноженное на Q равнялось 12 футам 40. И мы хотим что-то с этим сделать. Но это сделать намного проще, если мы осознаем, что левая часть — это просто производная по t. Ну, вы до 14 раз, и я объясню, что это значит через секунду.Итак, если вы возьмете производную от каждого из 40 умноженных на Q. Вы знаете, что должны использовать правило произведения и производную от сигналов. D. Q. D t Это путь, Рик. Вот и производная. Каждый из 40 — это от четырех до 14 раз. Хорошо, это твоя левая сторона. Это твоя правая сторона. И у нас есть сравнительно сложная производная в левой части. Поскольку мы хотели найти Q и Дика, U D t было бы намного проще, если бы мы могли упростить то, как это сделать здесь, взяв интеграл от обеих сторон, что избавит от этого ДДТ спереди.Итак, интегрируя науку, обе стороны. Мы можем просто посмотреть, что находится внутри производной здесь, так что это будет означать, что в 40 умноженных на Q, а затем на интеграл от 12 каждого из 40 DT, теперь интеграл от этого. Чтобы сэкономить время, используйте замену U, где вы равняете 40, и этот Интеграл выходит, чтобы съесть четыре раза три вас до 40 плюс спешка, и вы можете сделать это снова. Вы могли бы использовать замену u, но мы хотели найти то, что видим. И если вы помните, нам дали кубик, равный нулю.Итак, на правой стороне доски. Мы собираемся найти C. Итак, нам снова дали, что нулевой куб равен нулю с момента возникновения проблемы, и мы собираемся добавить ноль ко всему. Помните, что когда мы видим Q, это на самом деле функция Q. Of T. Итак, где бы мы ни видели T, мы направляемся к нулю вправо, и Q, равный нулю, Q станет просто двумя нулями, так как мы подключаемся, ноль для зубов равен любое время для нуля плюс C теперь e до четырех умножить на ноль — это просто съесть 20, что является одним милым из нуля, это ноль, то же самое здесь, трижды один плюс C.Но это означает, что их левая сторона — герой. В правой части также должно быть ноль, что означает, что c равно отрицательным трем. Итак, сейчас я скопирую это уравнение сюда с заменой C. У нас есть четыре тройки, умноженные на Q, равные в три раза каждому из четырех T минус три с тем, что у нас было на этом пересечении. Итак, три е к 40 минусовой полосе. А теперь, поскольку мы просто хотим найти Q, мы можем разделить на «съесть сорок», и это означает, что I e q равно трем. Он в 40 раз съел отрицательный за Т минус три е на отрицательный 14.Это просто означает один. Извините, это отрицательный знак. Проведите нас E к отрицательному 40. И я использовал закон распределения, чтобы оценить тройку, так что это Q. Это часть первая. А теперь нам нужно D. Q D T, и мы просто возьмем производную от этого и всего остального, так что производная от трех равна нулю. Таким образом, у нас будет отрицательный трижды отрицательный для E до отрицательного 40. Но это всего лишь 12. Съешьте отрицательный 14. Итак, наш ответ: q равно троекратному единице, когда мы едим, тогда я получаю 40 на DQ DT равно 12 e на отрицательный 40.Спасибо

Подключение проводов. Закон Ома для полной цепи. Электродвижущая сила

Электричество. Закон Ома. Последовательное и параллельное соединение проводов.

Если поместить изолированный проводник в электрическое поле, то сила будет действовать на свободные заряды q в проводнике. В результате в проводнике происходит кратковременное движение свободных зарядов. Этот процесс закончится, когда собственное электрическое поле зарядов, возникающих на поверхности проводника, полностью компенсирует внешнее поле.Результирующее электростатическое поле внутри проводника будет равно нулю.

Однако в определенных проводниках при определенных условиях может происходить непрерывное упорядоченное движение свободных носителей заряда.

Непрерывное упорядоченное движение зарядов называется электрическим током.

За направление электрического тока принято направление движения положительных свободных зарядов. Для существования электрического тока в проводнике необходимо создать в нем электрическое поле.

Количественным показателем электрического тока является ток I.

Скалярная физическая величина, равная отношению заряда Δq, переносимого через поперечное сечение проводника за интервал времени Δt, к этому интервалу времени, называется силой электрического тока. (рис. 1.7.1)

DIV_ADBLOCK15 «>

Природа внешних сил может быть разной. В гальванических элементах или батареях они возникают в результате электрохимических процессов, в генераторах постоянного тока Внешние силы возникают при движении проводников в магнитном поле.Источник тока в электрической цепи играет ту же роль, что и насос, который необходим для перекачивания жидкости в замкнутой гидросистеме. Под действием внешних сил электрические заряды перемещаются внутри источника тока против сил электростатического поля, так что в замкнутой цепи может поддерживаться постоянный электрический ток.

Когда электрические заряды движутся по цепи постоянного тока, внешние силы, действующие внутри источников тока, делают свою работу.

Физическая величина, равная отношению работы внешних сил Ast, когда заряд q перемещается от отрицательного полюса источника тока к положительному к величине этого заряда, называется источником электродвижущей силы (ЭДС):

DIV_ADBLOCK17 «>

Значение U12 обычно называется напряжением в цепи 1–2.В случае однородного участка напряжение равно разности потенциалов: U12 = φ1 — φ2.

Немецкий физик Г. Ом в 1826 году экспериментально установил, что сила тока I, протекающего через однородный металлический проводник (то есть проводник, в котором нет

Действует

внешних сил), пропорционально напряжению U на концах проводника:

где R = const.

Величина R называется электрическим сопротивлением. Проводник с электрическим сопротивлением называется резистором.Это соотношение выражает закон Ома для однородного сегмента цепи:

ток в проводнике прямо пропорционален приложенному напряжению и обратно пропорционален сопротивлению проводника.

В СИ единицей измерения электрического сопротивления проводов является ом (Ом). Сопротивление в 1 Ом имеет такой участок цепи, в котором при напряжении 1 В присутствует ток 1 А.

Проводники, подчиняющиеся закону Ома, называются линейными. Графическая зависимость силы тока I от напряжения U (такие графики называются вольт-амперными характеристиками, сокращенно ВАХ) представлена ​​прямой линией, проходящей через начало координат.

Для участка цепи, содержащего ЭДС, закон Ома записывается в следующей форме:

IR = U12 = φ1 — φ2 + ɛ = Δφ12 + ɛ.

Это соотношение обычно называют обобщенным законом Ома или законом Ома для неоднородного участка цепи.

На рис. 1.7.2 показывает замкнутую цепь постоянного тока. Участок цепи (cd) однородный.

Рисунок 1.7.2.

Замкнутая цепь постоянного тока.

Закон Ома для полной цепи: ток в полной цепи равен электродвижущей силе источника, деленной на сумму сопротивлений однородных и неоднородных участков цепи.

DIV_ADBLOCK19 «>

(R

Ток короткого замыкания — это максимальный ток, который может быть получен от этого источника с электродвижущей силой и внутренним сопротивлением r. Для источников с низким внутренним сопротивлением ток короткого замыкания может быть очень высоким и вызвать разрушение электрической цепи или источника. Например, в свинцовых аккумуляторах, используемых в автомобилях, ток короткого замыкания может составлять несколько сотен ампер. Особенно опасны короткие замыкания в осветительных сетях от подстанций (тысячи ампер).Чтобы избежать разрушительного воздействия таких больших токов, в схему включены предохранители или специальные автоматические выключатели.

В некоторых случаях, чтобы предотвратить опасные значения тока короткого замыкания, некоторое внешнее сопротивление подключается последовательно к источнику. Тогда сопротивление r равно сумме внутреннего сопротивления источника и внешнего сопротивления, и при коротком замыкании сила тока не будет чрезмерно большой.

Если внешняя цепь разомкнута, то Δφba = — Δφab = ɛ, т.е.е. разность потенциалов на полюсах разомкнутой батареи равна ее ЭДС.

Если внешнее сопротивление нагрузки R включено и через батарею протекает ток I, разность потенциалов на ее полюсах становится Δφba = — Ir.

На рис. 1.7.3 дано схематическое изображение источника постоянного тока с равной ЭДС и внутренним сопротивлением r в трех режимах: «холостой ход», работа на нагрузке и режим короткого замыкания (r.).

Рисунок 1.8.3.

Схематическое изображение источника постоянного тока: 1 — батарея разомкнута; 2 — батарея замкнута на внешнее сопротивление R; 3 — режим короткого замыкания.

Для измерения напряжений и токов в электрических цепях постоянного тока используются специальные приборы — вольтметры и амперметры.

Вольтметр предназначен для измерения разности потенциалов, приложенной к его клеммам. Он подключается параллельно к участку схемы, на котором измеряется разность потенциалов. Любой вольтметр имеет внутреннее сопротивление RB. Чтобы вольтметр не вносил заметного перераспределения токов при подключении к измеряемой цепи, его внутреннее сопротивление должно быть большим по сравнению с сопротивлением цепи, к которой он подключен.Для схемы, показанной на рис. 1.7 4 это условие записывается в виде: RB>> R1.

Это условие означает, что ток IB = Δφcd / RB, протекающий через вольтметр, намного меньше тока I = Δφcd / R1, протекающего через проверяемый участок цепи.

Поскольку внутри вольтметра нет внешних сил, разность потенциалов на его выводах по определению равна напряжению. Поэтому можно сказать, что вольтметр измеряет напряжение.

Амперметр

предназначен для измерения силы тока в цепи. Амперметр включен последовательно в разомкнутую цепь, так что весь измеряемый ток проходит через него. Амперметр также имеет внутреннее сопротивление RA. В отличие от вольтметра, внутреннее сопротивление амперметра должно быть достаточно малым по сравнению с общим сопротивлением всей цепи. Для схемы на рис. 1.7.4 Сопротивление амперметра должно удовлетворять условию RA.

чтобы при включении амперметра ток в цепи не менялся.

Измерительные приборы — вольтметры и амперметры — бывают двух типов: переключательные (аналоговые) и цифровые. Цифровые электроизмерительные приборы — это сложные электронные устройства. Обычно цифровые устройства обеспечивают более точные измерения.

Рисунок 1.7.4.

Включение амперметра (А) и вольтметра (В) в электрическую цепь

Последовательное и параллельное соединение проводов.

Проводники в электрических цепях можно соединять последовательно и параллельно.

При последовательном соединении проводов (рис. 1.8.1) ток во всех проводниках одинаков: I1 = I2 = I.

Рисунок 1.8.1.

Последовательное соединение проводов.

По закону Ома напряжения U1 и U2 на проводниках равны U1 = IR1, U2 = IR2.

Общее напряжение U на обоих проводниках равно сумме напряжений U1 и U2:

U = U1 + U2 = I (R1 + R2) = IR,

где R — электрическое сопротивление всей цепи.Это означает:

При последовательном соединении полное сопротивление цепи равно сумме сопротивлений отдельных проводов.

Этот результат действителен для любого количества последовательно соединенных проводов.

При параллельном подключении (рис. 1.8.2) напряжения U1 и U2 на обоих проводниках одинаковы: U1 = U2 = U.

Рисунок 1.8.2.

Параллельное соединение проводов.

Сумма токов I1 + I2, протекающих по обоим проводникам, равна току в неразветвленной цепи:

Этот результат следует из того факта, что в точках разветвления токов (узлы A и B) никакие заряды не могут накапливаться в цепи постоянного тока.Например, заряд IΔt течет к узлу A за время Δt, и заряд I1Δt + I2Δt течет из узла в то же время. Следовательно,

Запись по закону Ома:

где R — электрическое сопротивление всей цепи, получаем:

Когда проводники соединены параллельно, величина, обратная величине общего сопротивления цепи, равна сумме обратных сопротивлений параллельных проводов.

Этот результат действителен для любого количества параллельно соединенных проводов.

Формулы последовательного и параллельного соединения проводов позволяют во многих случаях рассчитать сопротивление сложной цепи, состоящей из множества резисторов. На рис. 1.8.3 является примером такой сложной схемы и указана последовательность вычислений.


Рисунок 1.8.3.

Расчет сопротивления сложной цепи. Сопротивления всех проводов указаны в омах (Ом)

Следует отметить, что не все сложные цепи, состоящие из проводов с разным сопротивлением, можно рассчитать по формулам для последовательного и параллельного соединения.На рис. 1.8.4 — это пример электрической цепи, которую невозможно рассчитать указанным выше методом.

Рисунок 1.8.4.

Пример электрической цепи, не сводящейся к комбинации последовательно соединенных и параллельно соединенных проводов

Урок №36-169 Подключение проводов. Закон Ома для полной цепи. Электродвижущая сила. D / C: 8,6; п.8.7; п.8.9

1. Подключение проводов.

1.1 последовательный — соединение , , у которого конец предыдущего проводника соединен с началом следующего.

С последовательным подключением: I 1 = я 2 (если ток постоянный, то за время t по любому участку проводника протекают одни и те же заряды)

У = U 1 + U 2 (работа электростатических сил при перемещении одиночного заряда в секциях 1 и 2 равна сумме работы в этих областях).

Эквивалентный проводник (сопротивление) — проводник, заменяющий группу проводников (сопротивлений) без изменения токов и напряжений на рассматриваемом участке цепи.

По закону Ома: U = ИК , г. тех. U 1 = IR 1; U 2 = IR 2;

IR = IR 1 + IR 2 = I (R 1 + R 2), т.е. R = R 1 + 2 или еще R =

Особый случай: R = nR , г.

При последовательном соединении эквивалентное сопротивление всей цепи равно сумме сопротивлений отдельных участков цепи. Поскольку I 1 = I 2; I 1 =; I 2 =; тогда U 1 = I 1 R 1 и U 2 = I 2 R 2, следовательно, =
Когда проводники соединены последовательно, напряжение, действующее на проводники, прямо пропорционально их сопротивлению.

Недостаток: при размыкании цепи одним из последовательно включенных потребителей ток пропадает по всей цепи (что на практике неудобно).

1,2 Параллельный — соединение , в котором начало жил соединено с одним узлом, а концы — с другим.

U = U 1 = U 2; I = I 1 = I 2 Закон: I = I 1 знак равно Я 2 =

, т.е. = + знак равно + или =

; q = q 1 + q 2

Проводимость всего соединения (всех вместе параллельно соединенных проводников) равна сумме проводимостей отдельных ветвей (каждого параллельно соединенного проводника).

Особый случай: R 1 = R 2 = … = R n, затем R =, где n — количество жил с одинаковым сопротивлением.

Из соотношений U 1 = U 2; U 1 =; U 2 = следует, что = — при параллельном соединении проводов токи в ответвлениях обратно пропорциональны их сопротивлениям.

Преимущество: если напряжение между узлами остается постоянным, то токи в ветвях независимы друг от друга.

2. Закон Ома для полной цепи

Полная цепочка содержит:

внешняя зона — потребителей тока, регулирующих, управляющих и т. Д.устройства с общим сопротивлением R

— внутренняя площадка — источник тока с ЭДС ε и с внутренним сопротивлением r (сопротивление, которым обладает источник электрической энергии, поскольку он является проводником, ток выделяет в нем тепло).

Рассмотрим замкнутую цепь, состоящую из внешней части, имеющей сопротивление R, и внутренней части — источника тока, сопротивление которого r

По закону сохранения энергии ЭДС источника тока равна к сумме падений напряжения на

внешней и внутренней частях цепи, так как при движении по замкнутой цепи заряд возвращается в исходное положение — в точку с таким же потенциалом (т.е. φ A = φ B): ε = ИК + Ir ,

где IR и Ir — падение напряжения на внешнем и внутреннем участках цепи. Следовательно, закон Ома для полной цепи:

3. ЭДС Действие внешних сил характеризуется физической величиной, называемой электродвижущей силой (ЭДС)

Электродвижущая сила в замкнутом контуре — это отношение работы внешних сил при движении заряда по цепи к заряду: ε =

Если на батарее записано 1.5 В, это означает, какие внешние силы (в данном случае heemick) заставляют работать 1,5 Дж при перемещении заряда на 1 C от одной батареи к другой. В замке не может быть постоянного тока цепная цепь, если ее нет в ней акт внешние силы т.е. нет ЭДС.

ЭДС, как и сила тока, является алгебраической величиной. Если ЭДС способствует перемещению

положительных зарядов в

выбранном направлении, то она считается положительной (ε> 0).Если ЭДС препятствует движению положительных зарядов в выбранном направлении, то она считается отрицательной (ε

Следует иметь в виду, что эту формулу можно использовать только тогда, когда ток течет внутри источника от отрицательного полюса к положительному. один, а во внешней цепи с плюса на минус.

3. Подключение источников электрической энергии к аккумулятору.

3.one. Последовательное подключение Полюс «+» предыдущего источника подключается к полюсу «-» последующего.Закон Ома для всей схемы при последовательном подключении. I =

3.2. Параллельное соединение «+» полюс подключается к одному выводу,

«-» полюс — к другому. Закон Ома для всей цепи при параллельном подключении

: I =

3.3 Смешанное подключение. Закон Ома для всей цепочки при смешанном соединении:

I =

Экзаменационные вопросы

А. 1,2 Ом В. 5,2 Ом В. 5 Ом

А.1,2 Ом B. 5,2 Ом В. 5 Ом

если R 1 = 2 Ом, R 2 = 3 Ом, R 3 = 4 Ом A. 1,2 Ом B. 5,2 Ом В. 5 Ом

31. Какая физическая величина определяется отношением работы, совершаемой внешними силами при движении заряда q по всей замкнутой электрической цепи, к величине этого заряда?

А. Ток. Б. Напряжение. Б. Электрическое сопротивление. G. Удельное электрическое сопротивление. D. Электродвижущая сила.

32. Какая из приведенных ниже формул выражает закон Ома для замкнутой цепи?

НО.Я = ; B.I =

; AT.IUΔt; G.P = Ui; D.ρ знак равно ρ 0 (1 + αt).

33. Источник тока с ЭДС 18 В имеет внутреннее сопротивление 30 Ом. Какое значение будет иметь ток при подключении к этому источнику резистора с электрическим сопротивлением 60 Ом? 0,6 A. B. 0,3 A. A. V. 0,2 A. G. 0,9 A.D. 0,4 A.

Задачи

1. Гальванический элемент с ЭДС 5,0 В и внутренним сопротивлением 0,2 Ом закорочен на проводник сопротивлением 40 Ом.0 Ом. Какое напряжение U на этом проводнике?

№ 2 В сеть с напряжением 220 В последовательно подключены две электролампочки

по 200 Ом каждая. Определите силу тока, проходящего через каждую лампу.

№ 3 Найдите полное сопротивление цепи, показанной на рисунке,

, если R 1 = 20 Ом, R 2 = R. 3 = R 4 = 15 Ом, R 5 = 3 Ом, R 6 = 90 Ом. .

№ 4. Дано четыре резистора по 60 Ом каждый. Нарисуйте схемы подключения всех четырех резисторов так, чтобы общее сопротивление было равным соответственно: 15, 45, 60, 80, 150 и 240 Ом.Возле каждой схемы написать расчет общего сопротивления.

№ 5. ЭДС источника электрической энергии 100 В. При внешнем сопротивлении 49 Ом ток в цепи

2 А. Найти падение напряжения внутри источника и его внутреннее сопротивление.

№ 6. Разность потенциалов на выводах открытого источника тока 4 В. Определите внутреннее сопротивление источника тока, если сопротивление внешнего участка цепи тока 4 Ом равно 0.8 А.

№ 7. Источник тока напряжением 220 В и внутренним сопротивлением 2 Ом замыкается проводником сопротивлением 108 Ом. Определите падение напряжения внутри источника тока.

№ 8. Определить ЭДС и внутреннее сопротивление источника тока, если ток в цепи 0,5 А при внешнем сопротивлении 3,9 Ом и ток при внешнем сопротивлении 1,9 Ом 1 А.

№ 9 . Определите силу тока при коротком замыкании аккумулятора с ЭДС 12 В, если при коротком замыкании на внешнее сопротивление 4 Ом ток в цепи составляет 2 А.Является ли схема наиболее актуальной?

№ 10. ЭДС источника тока 220 В, внутреннее сопротивление 1,5 Ом. Что нужно для того, чтобы сопротивление внешнего участка цепи было равным 4 А?

Закон Ома для участка цепи: сила тока I в электрической цепи прямо пропорционально напряжению U на концах участка и обратно пропорционально его сопротивлению р.

Формула закона: я =. Отсюда записываем формулы U = ИК и R = .

Рис.1. Сечение цепи Рис.2. Полная цепь

Закон Ома для полной цепи: сила тока I полная электрическая цепь равна электродвижущей силе (электродвижущей силе) источника тока E , деленной на полное сопротивление цепи ( R + r). Полное сопротивление цепи равно сумме сопротивлений внешней цепи. R и внутренний R источник тока. Формула закона I =

. На рис. 1 и 2 — схемы электрических цепей.

3. Последовательное и параллельное соединение проводов

Электрические проводники можно подключать. последовательно и параллельно . Смешанное соединение объединяет оба этих соединения.

Сопротивление, при включении вместо всех остальных проводников между двумя точками цепи ток и напряжение остаются неизменными, называется эквивалентным сопротивлением этих проводников.

Последовательное соединение

Последовательное называется соединением, в котором каждый проводник соединен только с одним предыдущим и одним последующим проводником.

Как следует из первых правил Кирхгофа Когда проводники соединены последовательно, сила электрического тока, протекающего через все проводники, одинакова (на основании закона сохранения заряда).

1. С последовательным подключением проводников (рис.1) сила тока во всех проводниках одинакова: I 1 = I 2 = I 3 = I

Рис. 1. Последовательное соединение двух проводов.

2. По закону Ома напряжения U 1 и U 2 на жилах равны У 1 = ИК 1 , г. U 2 = ИК 2 , г. U 3 = ИК 3 .

Напряжение при последовательном соединении проводов равно сумме напряжений на отдельных участках (проводниках) электрической цепи.

U = u1 + u2 + u3

Ом, напряжение U 1, У 2 на жилах равны У 1 = ИК 1 , г. U 2 = ИК 2 , г. Согласно второму правилу Кирхгофа, напряжение на всем участке:

U = U 1 + U 2 = ИК 1 + ИК 2 знак равно I (R ) 1 + R 2 ) = I · R. Получаем: R = R 1 + R 2

Общее напряжение U на проводниках равно сумме напряжений У 1 , г. U 2 , U 3 равно: U = U 1 + U 2 + U 3 = I · ( R 1 + R 2 + R 3 ) = ИК

где R ECV эквивалентно сопротивлению всей цепи.Отсюда: R ECV = R 1 + R 2 + R 3

При последовательном включении эквивалентное сопротивление цепи равно сумме сопротивлений отдельных участков цепи : R ECV = R 1 + R 2 + R 3 +…

Этот результат является удовлетворительным для любого числа последовательно соединенных проводников.

Из закона Омагодуета: в случае равенства сил тока при последовательном включении:

I = , г. I = . Отсюда знак равно или = , т.е. напряжения на отдельных участках цепи прямо пропорциональны сопротивлению участков.

При последовательном подключении n одинаковых проводников суммарное напряжение, равное произведению напряжения одного u 1 от их суммы n :

U ПОСЛЕ РОЖДЕНИЯ = n · U 1 . Аналогично для сопротивлений : R ПОСЛЕ РОЖДЕНИЯ знак равно n · R 1

При размыкании цепи одного из последовательно соединенных потребителей ток пропадает по всей цепи, поэтому последовательное подключение не всегда удобно на практике.

в последовательной цепи батарея имеет ЭДС, равную 1.5 В и внутреннее сопротивление r 1,0 Ом. Сопротивления R1, R2 и R3 составляют 5,0 Ом, 8,0 Ом и 6,0 Ом соответственно.

а) Сначала определим общее сопротивление в цепи. Поскольку проблема заключается в том, что мы имеем дело с последовательной цепью, это будет сумма всех отдельных сопротивлений.

R Всего = r + R 1 + R 2 + R 3 = 1,0 Ом + 5,0 Ом + 8,0 Ом + 6,0 Ом = 20,0 Ом.

Теперь мы можем рассчитать ток, протекающий в цепи, с помощью закона Ома.Закон Ома записывается в виде уравнения как V = IR, где V — напряжение, I — электрический ток, а R — сопротивление. Решая для тока, мы получаем I = V / R = (1,5 В) / (20,0 Ом) = 0,075 А.

Теперь, когда мы знаем ток, протекающий в цепи, мы можем применить закон Ома только к R1, чтобы найти напряжение. уронить. V = IR = (0,075 A) * (5,0 Ом) = 0,375 В.

б) Я лично никогда не имел дело с внутренним сопротивлением батарей и напряжениями на клеммах в своей курсовой работе, но я потратил некоторое время на поиск концепции и ее понимание чтобы помочь ответить на этот вопрос.При этом, если что-то из того, что я сказал, не имеет смысла, не стесняйтесь оставлять комментарий, и я посмотрю, чтобы убедиться, что я не сделал никаких ошибок. Напряжение на клеммах аккумулятора — насколько я понимаю — относится к напряжению, которое вы могли бы измерить, считывая разность напряжений на клеммах аккумулятора. По сути, это идеальное напряжение (ЭДС, указанная в задаче) за вычетом напряжения, потерянного из-за внутреннего сопротивления батареи.

Мы можем использовать закон Ома для расчета напряжения, теряемого на внутреннем сопротивлении батареи; мы знаем ток из части а) и знаем внутреннее сопротивление батареи, поэтому мы можем подключить их, чтобы найти падение напряжения:

В = IR = (0.075 A) * (1,0 Ом) = 0,075 В.

Тогда напряжение на клеммах — это просто ЭДС минус падение, или 1,5 В — 0,075 В = 1,425 В. Мы должны указать это как 1,4 В на основе правил для значащие цифры для сложения / вычитания.

c) Уменьшает напряжение на клеммах. Добавление резистора параллельно R2 уменьшит общее сопротивление цепи. Уменьшение общего сопротивления цепи приведет к увеличению тока. Таким образом, падение напряжения, вызванное аккумуляторной батареей, увеличится, а ЭДС останется прежней, поэтому напряжение на клеммах будет уменьшаться.

Сопротивление

и простые схемы — Физика колледжа, главы 1-17

Сводка

  • Объясните происхождение закона Ома.
  • Рассчитайте напряжения, токи или сопротивления по закону Ома.
  • Объясните, что такое омический материал.
  • Опишите простую схему.

Что движет током? Мы можем думать о различных устройствах, таких как батареи, генераторы, розетки и т. Д., Которые необходимы для поддержания тока. Все такие устройства создают разность потенциалов и условно называются источниками напряжения.Когда источник напряжения подключен к проводнику, он применяет разность потенциалов [латекс] \ boldsymbol {V} [/ latex], которая создает электрическое поле. Электрическое поле, в свою очередь, воздействует на заряды, вызывая ток.

Ток, протекающий через большинство веществ, прямо пропорционален приложенному к нему напряжению [латекс] \ boldsymbol {V} [/ latex]. Немецкий физик Георг Симон Ом (1787–1854) первым экспериментально продемонстрировал, что ток в металлической проволоке прямо пропорционален приложенному напряжению :

[латекс] \ boldsymbol {I \ propto V}.[/ латекс]

Это важное соотношение известно как закон Ома. Его можно рассматривать как причинно-следственную связь, в которой напряжение является причиной, а ток — следствием. Это эмпирический закон, подобный закону трения — явление, наблюдаемое экспериментально. Такая линейная зависимость возникает не всегда.

Если напряжение управляет током, что ему мешает? Электрическое свойство, препятствующее току (примерно такое же, как трение и сопротивление воздуха), называется сопротивлением RR размером 12 {R} {}. Столкновения движущихся зарядов с атомами и молекулами вещества передают энергию веществу и ограничивают ток.Сопротивление обратно пропорционально току, или

.

[латекс] \ boldsymbol {I \ propto} [/ latex] [латекс] \ boldsymbol {\ frac {1} {R}}. [/ Latex]

Таким образом, например, ток уменьшается вдвое, если сопротивление увеличивается вдвое. Комбинируя отношения тока к напряжению и тока к сопротивлению, получаем

[латекс] \ boldsymbol {I =} [/ latex] [латекс] \ boldsymbol {\ frac {V} {R}}. [/ Latex]

Это соотношение также называется законом Ома. Закон Ома в такой форме действительно определяет сопротивление определенных материалов.Закон Ома (как и закон Гука) не универсален. Многие вещества, для которых действует закон Ома, называются омическими. К ним относятся хорошие проводники, такие как медь и алюминий, и некоторые плохие проводники при определенных обстоятельствах. Омические материалы имеют сопротивление [латекс] \ boldsymbol {R} [/ latex], которое не зависит от напряжения [латекс] \ boldsymbol {V} [/ latex] и тока [латекс] \ boldsymbol {I} [/ latex]. Объект с простым сопротивлением называется резистором , даже если его сопротивление невелико.Единицей измерения сопротивления является Ом и обозначается символом [латекс] \ Омега [/ латекс] (греческое омега в верхнем регистре). Перестановка [latex] \ boldsymbol {I = V / R} [/ latex] дает [latex] \ boldsymbol {R = V / I} [/ latex], поэтому единицы сопротивления равны 1 Ом = 1 вольт на ампер:

[латекс] \ boldsymbol {1 \; \ Omega = 1} [/ latex] [латекс] \ boldsymbol {\ frac {V} {A}} [/ латекс]

На рисунке 1 показана схема простой схемы. Простая схема имеет один источник напряжения и один резистор. Можно предположить, что провода, соединяющие источник напряжения с резистором, имеют незначительное сопротивление, или их сопротивление можно указать в [латексе] \ boldsymbol {R} [/ латексе].

Рисунок 1. Простая электрическая цепь, в которой замкнутый путь для прохождения тока обеспечивается проводниками (обычно металлическими), соединяющими нагрузку с выводами батареи, представленной красными параллельными линиями. Зигзагообразный символ представляет собой единственный резистор и включает любое сопротивление в соединениях с источником напряжения.

Пример 1: Расчет сопротивления: автомобильная фара

Какое сопротивление проходит у автомобильной фары? 2.50 А течет при подаче на него 12,0 В?

Стратегия

Мы можем изменить закон Ома, как указано в [latex] \ boldsymbol {I = V / R} [/ latex], и использовать его для определения сопротивления.

Решение

Перестановка [latex] \ boldsymbol {I = V / R} [/ latex] и замена известных значений дает

[латекс] \ boldsymbol {R =} [/ latex] [латекс] \ boldsymbol {\ frac {V} {I}} [/ latex] [латекс] \ boldsymbol {=} [/ latex] [латекс] \ boldsymbol {\ frac {12.0 \; \ textbf {V}} {2.50 \; \ textbf {A}}} [/ latex] [latex] \ boldsymbol {= 4.80 \; \ Omega} [/ латекс]

Обсуждение

Это относительно небольшое сопротивление, но оно больше, чем хладостойкость фары. Как мы увидим в главе 20.3 Сопротивление и удельное сопротивление, сопротивление обычно увеличивается с температурой, поэтому лампа имеет меньшее сопротивление при первом включении и потребляет значительно больше тока во время короткого периода прогрева. 12 \; \ Omega} [/ latex] или более.{-5} \; \ Omega} [/ latex], а сверхпроводники вообще не имеют сопротивления (они неомичны). Сопротивление связано с формой объекта и материалом, из которого он состоит, как будет показано в главе 20.3 Сопротивление и удельное сопротивление.

Дополнительные сведения можно получить, решив [latex] \ boldsymbol {I = V / R} [/ latex], что дает

[латекс] \ boldsymbol {V = IR}. [/ Latex]

Это выражение для [latex] \ boldsymbol {V} [/ latex] можно интерпретировать как падение напряжения на резисторе, вызванное протеканием тока [latex] \ boldsymbol {I} [/ latex].Фраза [latex] \ boldsymbol {IR} [/ latex] drop часто используется для обозначения этого напряжения. Например, у фары в Примере 1 падение [latex] \ boldsymbol {IR} [/ latex] составляет 12,0 В. Если напряжение измеряется в различных точках цепи, будет видно, что оно увеличивается на источнике напряжения и уменьшается. на резисторе. Напряжение аналогично давлению жидкости. Источник напряжения подобен насосу, создающему перепад давления, вызывающему ток — поток заряда. Резистор похож на трубу, которая снижает давление и ограничивает поток из-за своего сопротивления.Здесь сохранение энергии имеет важные последствия. Источник напряжения подает энергию (вызывая электрическое поле и ток), а резистор преобразует ее в другую форму (например, тепловую энергию). В простой схеме (с одним простым резистором) напряжение, подаваемое источником, равно падению напряжения на резисторе, поскольку [latex] \ boldsymbol {PE = q \ Delta V} [/ latex], и то же самое qq размер 12 {q} {} проходит через каждую. Таким образом, энергия, подаваемая источником напряжения, и энергия, преобразуемая резистором, равны.(См. Рисунок 2.)

Рисунок 2. Падение напряжения на резисторе в простой цепи равно выходному напряжению батареи.

Установление соединений: сохранение энергии

В простой электрической цепи единственный резистор преобразует энергию, поступающую от источника, в другую форму. Здесь о сохранении энергии свидетельствует тот факт, что вся энергия, подаваемая источником, преобразуется в другую форму только с помощью резистора. Мы обнаружим, что сохранение энергии имеет другие важные применения в схемах и является мощным инструментом анализа схем.

Исследования PhET: закон Ома

Посмотрите, как уравнение закона Ома соотносится с простой схемой. Отрегулируйте напряжение и сопротивление и посмотрите, как изменяется ток по закону Ома. Размеры символов в уравнении изменяются в соответствии с принципиальной схемой.

Рисунок 3. Закон Ома
  • Простая схема — это схема , в которой есть один источник напряжения и одно сопротивление.
  • Одно из утверждений закона Ома устанавливает связь между током [латекс] \ boldsymbol {I} [/ latex], напряжением [латекс] \ boldsymbol {V} [/ latex] и сопротивлением [латекс] \ boldsymbol {R} [/ latex] в простой схеме быть [latex] \ boldsymbol {I = \ frac {V} {R}} [/ latex].
  • Сопротивление имеет единицы измерения в омах ([латекс] \ boldsymbol {\ Omega} [/ latex]), относящиеся к вольтам и амперам с помощью [латекса] \ boldsymbol {1 \; \ Omega = 1 \; \ textbf {V} / \ textbf {A}} [/ латекс].
  • На резисторе наблюдается падение напряжения или [латекс] \ boldsymbol {IR} [/ latex], вызванное протекающим через него током, определяемое выражением [latex] \ boldsymbol {V = IR} [/ latex].

Концептуальные вопросы

1: Падение [латекс] \ boldsymbol {IR} [/ latex] на резисторе означает изменение потенциала или напряжения на резисторе.Изменится ли ток при прохождении через резистор? Объяснять.

2: Как падение [латекс] \ boldsymbol {IR} [/ latex] в резисторе похоже на падение давления в жидкости, протекающей по трубе?

Задачи и упражнения

1: Какой ток протекает через лампочку фонаря на 3,00 В, когда его горячее сопротивление составляет [латекс] \ boldsymbol {3.60 \; \ Omega} [/ latex]?

2: Рассчитайте эффективное сопротивление карманного калькулятора с 1.Аккумулятор 35 В и через который протекает 0.200 мА.

3: Каково эффективное сопротивление стартера автомобиля, когда через него проходит 150 А, когда автомобильный аккумулятор подает на двигатель 11,0 В?

4: Сколько вольт подается для работы светового индикатора DVD-плеера с сопротивлением [латекс] \ boldsymbol {140 \; \ Omega} [/ latex], учитывая, что через него проходит 25,0 мА?

5: (a) Найдите падение напряжения в удлинителе с символом [латекс] \ bold {0.0600- \ Omega} [/ latex] сопротивление, через которое протекает ток 5,00 А. (б) В более дешевом шнуре используется более тонкая проволока, и его сопротивление составляет [латекс] \ boldsymbol {0,300 \; \ Omega} [/ латекс]. Какое в нем падение напряжения при протекании 5.00 А? (c) Почему напряжение на любом используемом приборе снижается на эту величину? Как это повлияет на прибор?

6: ЛЭП подвешена к металлическим опорам со стеклянными изоляторами, имеющими сопротивление [латекс] \ boldsymbol {1,00 \ times 10 ^ 9 \; \ Omega} [/ латекс].Какой ток протекает через изолятор при напряжении 200 кВ? (Некоторые линии высокого напряжения — постоянного тока.)

Глоссарий

Закон Ома
— эмпирическое соотношение, указывающее, что ток I пропорционален разности потенциалов V , V ; его часто записывают как I = V / R , где R — сопротивление
сопротивление
электрическое свойство, препятствующее току; для омических материалов это отношение напряжения к току, R = V / I
Ом
единица сопротивления, равная 1Ω = 1 В / A
омический
вид материала, для которого действует закон Ома
простая схема
схема с одним источником напряжения и одним резистором

Решения

Задачи и упражнения

1: 0.{-2} \; \ Omega} [/ латекс]

5: (а) 0,300 В

(б) 1,50 В

(c) Напряжение, подаваемое на любой используемый прибор, снижается, поскольку общее падение напряжения от стены до конечного выхода прибора является фиксированным.

alexxlab

Добавить комментарий

Ваш адрес email не будет опубликован. Обязательные поля помечены *